Download as pdf or txt
Download as pdf or txt
You are on page 1of 229

Dr.

Wael Abboud (Foot and Ankle OB)

‫بسم اهلل الرحمن الرحيم‬


Foot and Ankle OB(120)
1- Which of the following descriptions of muscle activity
during normal gait is correct?
1. Gastrocnemius-soleus contracts eccentrically during
heel strike
2. Gastrocnemius-soleus contracts concentrically during
heel strike
3. Gastrocnemius-soleus contracts concentrically during
swing phase
4. Tibialis anterior contracts concentrically during toe-off
5. Tibialis anterior contracts eccentrically during heel
strike

PREFERRED RESPONSE ▼ 5
CORRECT

One stride (heel strike to heel strike of one leg) of normal gait has
been divided into the stance (62%) and swing (38%) phases. The
stance phase is further divided into heel strike, foot flat, and toe
off. Proper gait requires coordinated contraction of the leg
muscles. The tibialis anterior (TA) muscle fires eccentrically at heel
strike to lower the foot to the ground, while the gasto-soleus (GS)
complex is dormant. The TA then relaxes, while the GS
eccentrically contracts as the body’s weight is transferred forward
over the foot during foot-flat. As the foot propels the body forward
during toe-off, the GS contracts concentrically, while the TA
remains dormant. As swing commences, the TA then fires
concentrically producing dorsiflexion to clear the foot over the
ground while the GS relaxes. Certain conditions like cerebral palsy
result in improper firing of the muscles during the gait cycle,
resulting in altered gait mechanics.
Dr.Wael Abboud (Foot and Ankle OB)

2- Which of the following does NOT occur during the normal


push-off stance of the gait cycle?

1. subtalar joint inversion


2. transverse tarsal joint locks
3. external rotation of the lower extremity
4. tightening of the plantar fascia
5. loosening of the spring ligament

PREFERRED RESPONSE ▼ 5
CORRECT

During the normal gait cycle, the foot changes from a flexible
structure at heel strike to a rigid structure at toe-off. The
mechanisms that bring about this conversion are (1) tightening of
the plantar aponeurosis, (2) progressive external rotation of the
lower extremity, which begins at the pelvis and is passed distally
across the ankle joint to the subtalar joint, and (3) stabilization of
the transverse tarsal joint which results from progressive inversion
of the subtalar joint. Van Boerum and Sangeorzan cite triceps
surae insufficiency, obesity, posterior tibial tendon dysfunction, or
ligamentous laxity in the spring ligament or other ligaments as
possible causes for flatfoot.

3- Which of the following structures is the primary site of


force transfer between the hindfoot and forefoot during the
stance phase of gait?
QID: 17

1. Plantar aponeurosis
2. Achilles tendon
3. Lisfranc ligament
4. Posterior tibial tendon
5. Anterior tibial tendon
Dr.Wael Abboud (Foot and Ankle OB)

PREFERRED RESPONSE ▼ 1
CORRECT

During ambulation, the transverse tarsal joints are parallel during


the heel strike phase of stance, when the calcaneus is in eversion,
allowing the midfoot to be supple for shock absorption of the
body's weight. During the push-off phase of stance the subtalar
joint moves to inversion, which makes the foot rigid so that it can
create a rigid lever arm for propulsion. The referenced study by
Erdemir et al found that plantar aponeurosis forces gradually
increased during stance and peaked in late stance, with maximum
tension reaching 96% of total body weight. The authors concluded
the plantar aponeurosis transmits large forces between the
hindfoot and forefoot during the stance phase of gait.

4- The primary antagonist of the anterior tibial tendon is


innervated by which of the following nerves?

1. Superficial peroneal nerve


2. Deep peroneal nerve
3. Tibial nerve
4. Posterior tibial nerve
5. Sural nerve

PREFERRED RESPONSE ▼ 1
CORRECT

The primary antagonist of the anterior tibial tendon is the peroneus


longus, which is innervated by the superficial peroneal nerve. The
peroneus longus (PL) and tibialis anterior (TA) are antagonists to
each other as the PL plantarflexes and everts, while the TA
dorsiflexes and inverts the foot. The cited cadaveric study by Silver
Dr.Wael Abboud (Foot and Ankle OB)

found that the plantarflexors of the ankle were six times as strong
as the dorsiflexors.

5-Which of the following best describes the physical


examination test demonstrated in Figure A?

1. Silfverskiöld test used to differentiate gastrocnemius


tightness from achilles tendon contracture
2. Thompson test used to differentiate soleus tightness
from achilles tendon contracture
3. Coleman test used to differentiate soleus tightness from
achilles tendon contracture
4. Silfverskiöld test to differentiate soleus tightness from
achilles tendon contracture
5. Thompson test to differentiate gastrocnemius tightness
from achilles tendon contracture

PREFERRED RESPONSE ▼ 1
Dr.Wael Abboud (Foot and Ankle OB)

The Silfverskiöld test differentiates gastrocnemius tightness from


an achilles tendon contracture by evaluating ankle dorsiflexion with
the knee extended and then flexed. Increased ankle dorsiflexion
with knee flexion indicates gastrocnemius tightness. This occurs
because the gastrocnemius relaxes with knee flexion as the
muscle spans the knee joint and the soleus does not. If there is no
difference in dorsiflexion with flexion of the knee, then an achilles
tendon contracture is present.

6-A 54-year-old male with Protein C deficiency sustained a


stroke 3 months ago with subsequent left sided weakness.
The patient's skin is intact with no sign of ulcerations or
callosities. His left foot is developing a varus deformity
secondary to a spastic tibialis anterior. His ankle has a 5
degree plantar flexion deformity due to a gastrocnemius
contracture which improves with knee flexion (positive
Silverskiold test). The patient has an intrinsic minus foot with
supple claw toes present. Which of the following is the most
appropriate next step in management?

1. Gastrocnemius fascia lengthening (Strayer) procedure


2. Split anterior tibial tendon transfer (SPLATT)
3. Flexor hallucis longus tendon transfer to the dorsum of
the foot and release of the flexor digitorum longus and brevis
tendons at the base of each toe
4. Fractional lengthening of the tibialis posterior
5. Ankle foot orthosis (AFO) with physical therapy

PREFERRED RESPONSE ▼ 5
CORRECT

The key to this patient's presentation is the occurrence of the


stroke only 3 months ago. Miller Review states at least a 6 month
delay is required and the review article by Younger and Hansen
Dr.Wael Abboud (Foot and Ankle OB)

state a delay of 18 to 24 months between cerebral injury and


reconstructive surgery is advisable because of the possibility of
various degrees of functional recovery. The critcial components to
nonoperative management include AFO fitting, early intervention
with physical therapy, stretching and strengthening, and
maintenance of joint range of motion. Phenol or botox injections
can also be useful nonsurgical adjuvants.
Equinovarus posturing is the most common ankle-foot abnormality
following a stroke. The equinus component is caused by
overactivity of the gastrocnemius-soleus complex. The varus is
due to relative overactivity of the tibialis anterior, with contributions
from the tibialis posterior, FHL, and FDL. Each of the surgical
options are reasonable treatments for this patient's presentation if
there had been an adequate interval between the patient's cerebral
insult and he had failed conservative management.

7-A 35 year-old man is referred to you for left foot pain after
falling from a bike and he brings a MRI shown in Figure A. The
injured structure demonstrated in the MRI will most likely lead
to which of the following deformities if left untreated?

1. syndesmotic widening
2. flatfoot deformity
3. widening of the 1st and 2nd tarsometatarsal joints
Dr.Wael Abboud (Foot and Ankle OB)

4. hallux valgus deformity


5. talar tilt deformity

PREFERRED RESPONSE ▼ 2
CORRECT

Figure A demonstrates a disrupted calcaneonavicular ligament


(Spring ligament). The ability of the medial longitudinal arch to
prevent flatfoot deformity depends on the dynamic support of the
posterior tibial tendon, the static support of ligaments and capsule
(including the Spring ligament), and the manner in which the tarsal
bones interlock. Gadzag retrospectively found that 18 of 22
patients undergoing surgery for flatfoot deformity had varying
levels of injury to the spring ligament but their study design
prevented them from concluding whether ligament repair was
beneficial. Illustration A contains an MRI of a normal Spring
ligament shown with the green arrow compared to the a disrupted
Spring ligament shown with the red arrow.
Dr.Wael Abboud (Foot and Ankle OB)

8-Nerve injury is the most common complication of ankle


arthroscopy. Which of the following nerves is most commonly
injured with use of the anterolateral portal?

1. Superficial peroneal nerve


2. Deep peroneal nerve
3. Sural nerve
4. Saphenous nerve
5. Tibial nerve
Dr.Wael Abboud (Foot and Ankle OB)

PREFERRED RESPONSE ▼ 1
CORRECT

Nerve injury is the most common complication of ankle


arthroscopy. The superficial peroneal nerve is the most commonly
injured nerve and is at risk with use of the anterolateral portal. The
1996 study by Ferkel et al is a retrospective review looking at
complications associated with ankle arthroscopy. They found an
overall complication rate of 9.0%, of these 4.4% were neurological
complications. Of the 27 cases of nerve injury, the superficial
peroneal nerve was injured in 15 cases, the sural nerve in 6, the
saphenous nerve in 5, and the deep peroneal nerve in 1. All nerve
injuries occurred through direct injury by portal or distractor pin
placement. The 2001 article by Ferkel et al is a review article that
discusses complications associated with ankle arthroscopy and
strategies to prevent them.

9-A patient with subtalar and tibiotalar arthritis underwent the


surgery shown in Figure A. The patient now complains of
numbness on the plantar/lateral aspect of his foot including
the 4th and 5th toes. Which nerve was most likely injured?
Dr.Wael Abboud (Foot and Ankle OB)

1. Lateral plantar
2. Medial plantar
3. Sural
4. Superficial Peroneal
5. Deep Peroneal

PREFERRED RESPONSE ▼ 1
CORRECT

The lateral plantar nerve innervates the plantar skin covering the
lateral half of the fourth toe and the entire fifth toe. This nerve also
provides motor innervation to many of the deep muscles in the
foot. Both references concluded that the optimal insertion site is in
close proximity to many neurovascular structures. They
recommend doing a careful dissection of the heel to avoid damage
to any structure(s). The structure that Flock et al found to be at risk
specifically was the nerve to the abductor digiti quinti, which is a
branch of the lateral plantar nerve.
Dr.Wael Abboud (Foot and Ankle OB)
Dr.Wael Abboud (Foot and Ankle OB)

10-When performing an ankle fusion, the foot should be in:

1. 0 degrees dorsiflexion/plantarflexion, 0-5 degree


hindfoot valgus, 5-10 degree external rotation
2. 0 degrees dorsiflexion/plantarflexion, 0-5 degrees
hindfoot valgus, 0 degrees external rotation
3. 10 degrees dorsiflexion, 0-5 degrees hindfoot valgus, 5-
10 degrees external rotation
4. 0 degrees dorsiflexion/plantarflexion, 20 degrees
hindfoot valgus, 5-10 degrees external rotation
5. 10 degrees plantarflexion, 0-5 degrees hindfoot valgus,
5-10 degrees external rotation

PREFERRED RESPONSE ▼ 1
CORRECT
Dr.Wael Abboud (Foot and Ankle OB)

Buck et al in a retrospective analysis of 19 patients who underwent


ankle fusion at 10 years follow-up found that: “The patterns of gait
showed that a valgus position of the arthrodesis is more
advantageous and provides more normal gait, particularly on
uneven ground. To attain more normal function of the knee and
improve performance on rough ground, the optimum position of
arthrodesis of the ankle appears to be neutral flexion, slight (zero
to 5 degrees) valgus angulation, and approximately 5 to 10
degrees of external rotation.”

Plantarflexion (#5) is never that answer as the goal is to have a


plantigrade foot. Too much dorsiflexion (#3) or valgus (#4) alter the
gait and place increased stress on the knee.

11-A 57-year-old administrative assistant complains of pain


over the bunion on her right foot. Physical exam is notable for
tenderness over the medial prominence of the first
metatarsophalangeal joint and hypermobility of the first ray.
Shoe modifications have failed to provide relief. A clinical
photograph and radiograph are provided in figures A and B.
Surgical treatment with metatarsocuneiform arthrodesis is
chosen. Each of the following are associated with a better
clinical outcome EXCEPT:
Dr.Wael Abboud (Foot and Ankle OB)

1. Multiple screw fixation across the metatarsocuneiform


arthrodesis
2. Augmentation of the metatarsocuneiform arthrodesis
with bone grafting
3. Dorsiflexion unloading of the first metatarsal
4. Correction of the first intermetatarsal angle
5. Failure of conservative treatments prior to surgery

PREFERRED RESPONSE ▼ 3
CORRECT

The clinical photograph and radiograph demonstrate hallux valgus


with a large first intermetatarsal angle. Metatarsus primus varus
and hypermobility of the first ray are both indications for including
metatarsocuneiform arthrodesis in the surgical correction of hallux
valgus. Anatomic plantarflexion of the first metatarsal is crucial to
prevent loading of the lesser metatarsals following surgery.
Myerson et al reports the results of 53 patients treated with
metatarsocuneiform arthrodesis for hallux valgus and metatarsus
primus varus. They found that 92% acheived clinical satisfaction
despite numerous complications including 7 superficial pin tract
Dr.Wael Abboud (Foot and Ankle OB)

infections, 3 dorsal bunions, 7 nonunions, 1 hallux varus, and 3


neuromas of the deep peroneal nerve. Only 1 of the complications
needed secondary surgery. Sangeorzan et al reports the results of
metatarsocuneiform arthrodesis in 33 patients with hallux valgus
and a hypermobile first ray and 7 patients with previous failed
bunion surgery. Successful union occured in 90%, and 75% were
considered successful clinical results. Best results were acheived
with bone grafting, multiple screw fixation, and accurate
plantarflexion of the first metatarsal.

12-A 67-year old female presents with the bilateral foot


deformity shown in Figures A and B. All of the following
contribute to the risk of recurrence after surgery EXCEPT:

1. Resection of the lateral sesamoid


2. Lack of lateral capsular release
3. Lack of medial metatarsophalangeal joint capsule
closure
4. Use of an Akin procedure alone for a moderate to severe
Dr.Wael Abboud (Foot and Ankle OB)

deformity
5. Undercorrection of the widened 1-2 intermetatarsal (IMA)
angle

PREFERRED RESPONSE ▼ 1
CORRECT

The patient has bilateral hallux valgus. Resection of the lateral


sesamoid would likely lead to hallux varus and not hallux valgus
recurrence. Akin procedure is a closing wedge osteotomy of the
proximal phalanx when the proximal phalangeal articular angle
>10° but does not correct hallux valgus well. Metatarsal
osteotomies, such as the Chevron, are used to bring the
metatarsal shaft laterally to reduce the IMA but undercorrection of
the IMA would likely lead to hallux valgus recurrence. Lateral soft
tissue release and medial capsular reefing can be used to correct
hallux valgus in combination with bony procedures, but its
important to avoid an extensive lateral capsular release to
minimize the risk of metatarsal head osteonecrosis.

13-A 55-year-old female attorney complains of progressive


pain and deformity of the great toe. The pain is localized to a
prominence of the medial metatarsal head. A clinical image is
provided in figure A. Which of the following best describes
the hallux deformity?
Dr.Wael Abboud (Foot and Ankle OB)

1. Valgus and supination


2. Valgus and pronation
3. Valgus and hyperextension
4. Varus and supination
5. Varus and pronation

PREFERRED RESPONSE ▼ 2
CORRECT

Bunions are a common deformity of the great toe that is most


prevalent in the aging female cohort. The etiology is multi-factorial
including genetic predisposition and chronic exposure to narrow
toe box shoes. The static soft tissue constraints attenuate medially
and contract laterally, and the plantar dynamic flexors deviate
laterally pulling the great toe into valgus and pronation. Illustration
A is an example of this deformity. Coughlin summarizes the
evaluation, diagnosis, and treatment of hallux valgus.

14-A 57-year-old female underwent surgery for severe hallux


rigidus. Postoperative radiographs are shown in Figure A.
One year later she complains of pain at the 2nd metatarsal
head and her exam shows a plantar callosity under the 2nd
metatarsal head. What procedure should have been combined
with her initial operation to prevent this outcome?
Dr.Wael Abboud (Foot and Ankle OB)

1. Second metatarsalphalangeal joint arthrodesis


2. Second metatarsal osteotomy (Weil) with extensor
tendon and dorsal capsular release
3. Flexor to extensor tendon transfer (Girdlestone-Taylor)
4. Second metatarsal osteotomy (Helal) with extensor
tendon and dorsal capsular release
5. Second metatarsal head resection with extensor tendon
and dorsal capsular release

PREFERRED RESPONSE ▼ 2
CORRECT

The second metatarsal osteotomy (Weil), is an intra-articular


osteotomy that achieves longitudinal decompression through
shortening and will correct the longer 2nd metatarsal in relation to
the first metatarsal. The first metatarsal bears half the weight of the
forefoot and this patient's plantar callosity under the 2nd
metatarsal head is clinical evidence of abnormal pressure transfer
due to the longer 2nd metatarsal (seen in Figure A). The foot tripod
can also be altered by hallux valgus as the proximal phalanx
moves into valgus, the splay between the first and second rays
increases (IM angle), the metatarsal moves into varus and
elevates, and weight-bearing is transferred from the 1st metatarsal
head to the the 2nd metatarsal head. A shortening osteotomy with
extensor tendon and dorsal capsular release is the most
appropriate option listed to address the second MTP
metatarsalgia.
The Weil osteotomy of the metatarsal head is preferable to the
Helal osteotomy of the metatarsal shaft to correct metatarsalgia
due to higher rates of nonunion and pseudarthrosis with shaft
Dr.Wael Abboud (Foot and Ankle OB)

osteotomies as demonstrated in the article by Trnka et al.


Illustration A shows the difference between the Weil osteotomy
(osteotomy B) and the Helal osteotomy (osteotomy A).

15-Which of the following clinical scenarios regarding hallux


valgus could be appropriatley treated with a modified McBride
procedure?

1. 35-year-old female with a 20 degree HVA, a 11 degree


IMA, and an incongruent 1st MTP joint
2. 40-year-old male with a 30 degree HVA, and a 15 degree
IMA, and a congruent 1st MTP joint
3. 70-year-old female with a 35 degree HVA, and a 13
degree IMA with a hypermobile 1st ray
4. 65-year-old female with a 25 degree HVA, a 14 degree
IMA, and severe hallux rigidus
5. 85-year old minimally ambulatory male with a 45 degree
HVA, and a 20 degree IMA

PREFERRED RESPONSE ▼ 1
CORRECT

The Modified McBride procedure is indicated in patients 30-50


years old with an incongruent joint, a HVA less than 25 degrees,
and an IMA deformity less than 15 degrees. This soft tissue
procedure should be avoided in moderate or severe hallux valgus
deformity due to the increased risk of recurrence. Surgical
technique includes release of adductor hallucis, transverse
metatarsal ligament, and lateral capsule combined with excision of
medial eminence and plication of the capsule medially. In patients
with moderate hallux valgus deformity (HVA 26-40 deg, IMA 13-15
deg) a proximal osteotomy should be performed. In patients with
moderate deformity and a hypermobile 1st ray, a Lapidus
Dr.Wael Abboud (Foot and Ankle OB)

procedure (includes 1st TMT fusion) should be considered. A MTP


arthodesis is indicated when concomitant severe hallux rigidus is
present, and a Keller (includes partial resection of proximal
phalanx) procedure should be considered in an elderly, low
demand individual with severe deformity. Illustration A is an AP
radiograph of the foot demonstrating the technique used in
measuring the HVA and IMA.

16-A 34-year-old woman presents with right foot pain and and
a callus over the 1st metatarsalphalangeal joint. A clinical
image is shown in Figure A. Accomodative shoewear has
failed to relieve symptoms. Images displaying key
radiographic angles in the evaluation of this disorder are
shown in Figures B and C. Which of the following operative
procedures is most appropriate for this deformity?
Dr.Wael Abboud (Foot and Ankle OB)

1. Closing wedge osteotomy of the proximal phalanx (Akin)


2. Distal soft-tissue release
3. Distal metatarsal osteotomy
Dr.Wael Abboud (Foot and Ankle OB)

4. Medial eminence resection and exostectomy (Silver)


5. Proximal metatarsal osteotomy

PREFERRED RESPONSE ▼ 5
CORRECT

This patient has a moderate hallux valgus deformity as shown by


the intermetatarsal angle of 15 degrees shown in Figure B, and the
hallux valgus angle of 40 degrees shown in Figure C. Proximal
metatarsal osteotomy is the only procedure of the available options
in isolation appropriate for this amount of deformity.

The review article by Jones et al describes multiple methods of


proximal osteotomies including crescentic, chevron, and oblique
osteotomies. Overcorrection of IMA can lead to hallux varus and
dorsiflexion at osteotomy can lead to transfer metatarsalgia.
Double osteotomies can also be utilized for moderate deformities
with Jones et al describing 92% satisfaction with a combined Akin
and scarf osteotomy (Illustration A).
Illustration B demonstrates a hallux valgus deformity with a
congruent MTP joint, as is also the case for the patient in this
question. A distal soft-tissue release, including a release of the
lateral structures, would be an appropriate procedure combined
with a proximal metatarsal osteotomy if the MTP joint was
incongruent.
Dr.Wael Abboud (Foot and Ankle OB)

17-A shoe orthotic with a Morton's extension is indicated for


which of the following conditions?

QID: 3179

1. Hallux valgus
2. Hallux rigidus
3. 2nd metatarsophalangeal joint synovitis
4. Jones fracture
5. Hammertoe deformity

PREFERRED RESPONSE ▼ 2
CORRECT

A Morton's extension is a stiff insert with a rigid extension under


the 1st ray. This extension is used to conservatively treat
conditions such as hallux rigidus, as it limits extension of the 1st
MTP joint during the push-off phase of gait. The first referenced
article by Shereff et al is a review of hallux rigidus, including
diagnosis, staging, and treatment. The second referenced article
Dr.Wael Abboud (Foot and Ankle OB)

by Sammarco et al is a review of the orthotic management of


different problems around the hallux, including rigidus. Illustration
A is a photo of a commonly available Morton's extension.

18-A 45-year-old construction worker reports pain in the first


toe with the maneuver found in Figure A. The radiographs in
Figure B and C reveals mild osteoarthritis of the first
metatarsophalangeal joint and a dorsal osteophyte. Orthotics
have failed to provide relief. What surgical procedure is the
most appropriate next step in management?
Dr.Wael Abboud (Foot and Ankle OB)

1. Modified chevron osteotomy


2. Metatarsal dorsal oblique osteotomy (Helal osteotomy)
3. Metatarsal plantar oblique osteotomy (Weil osteotomy)
4. Cheilectomy
5. Proximal phalanx closing wedge osteotomy (Moberg
osteotomy)

PREFERRED RESPONSE ▼ 4
CORRECT

Cheilectomy, as shown in Illustration A, will relieve dorsal


impingement pain associated with Grade 1-2 MTP arthritis.
Cheilectomy is offered after conservative management has failed
Dr.Wael Abboud (Foot and Ankle OB)

(including NSAIDs and orthotics such as a rigid Morton's extension


foot orthosis). Moberg osteotomies are closing wedge osteotomies
of the proximal phalanx that remove a dorsal wedge of bone to
improve the functional range of motion arc of the MTP. A Weil
osteotomy is an intra-articular osteotomy that achieves longitudinal
decompression through shortening. In a Weil osteotomy the
metatarsal (MT) is exposed and the direction of shortening runs
mostly parallel to the plantar aspect of the foot. The Weil
osteotomy is fixed by means of a screw running perpendicular to
the osteotomy line. The Helal osteotomy has no form of fixation
and is made more proximally. Illustration B compares the Helal (A)
and Weil (B) osteotomies and they are often used as shortening
procdures in the treatment of MTP dislocations. Arthrodesis is
treatment of choice following failed cheilectomy or where
advanced degenerative changes are present.

The review article by Sammarco and Nichols discuss orthotic


options for hallux rigidus, hallux valgus, sesamoiditis, and 1st ray
hypermobility.

The article by Yee and Lau is a current concepts review of hallux


rigidus.

19-An 80-year-old female presents with a painful great toe


MTP joint. She is minimally ambulatory, and has failed
Dr.Wael Abboud (Foot and Ankle OB)

conservative treatment. A physical exam reveals a


plantarflexed great toe which does not allow for comfortable
shoe wear. AP and lateral radiographs of the foot are shown
in Figures A and B. What is the most appropriate surgical
treatment that will allow for the fastest rehabilitation?

1. Implantation of a double-stem silicone implant


2. Dorsiflexion osteotomy (Moberg) of the proximal
phalanx
3. resection arthroplasty (Keller) along with removal of
osteophytes
4. Cheilectomy
5. Arthrodesis of the first metatarsophalangeal joint

PREFERRED RESPONSE ▼ 3
CORRECT

Yee et al provide a complete review of hallux rigidus which outlines


the clinical presentation and treatment options, including
indications and the evidence to support them. They state that a
resection arthroplasty (Keller) is the best option for the treatment of
advanced hallux rigidus in older, low demand patients. Silicone
implants are less desirable due to the potential for synovitis and
implant failure, and salavage operations are difficult. A Moberg
dorsiflexion osteotomy of the proximal phalanx is indicated in the
rare situation where there is symptomatic hyperextension of the
interphalangeal joint due to a stiff, but pain-free MTP joint.
Dr.Wael Abboud (Foot and Ankle OB)

Cheilectomy alone would not address the significant joint


degeneration, and an arthrodesis is an accepted surgical option for
advanced-stage hallux rigidus, particularly in younger and more
active patients. Gibson et al performed a RCT to evaluate
outcomes after first MTPJ arthrodesis and replacement
arthroplasty. Outcomes after arthrodesis were superior, and there
was an unacceptably high incidence of loosening of the
arthroplasty components. Illustration A depicts a standard Keller
procedure.

20-You are seeing a 60-year-old male for pain in his great toe
that has increased in severity over the past year despite the
use of an insole with a morton extension. His
plantar/dorsiflexion range of motion is limited to a 35 degree
arc with pain at the extremes of motion. Radiographs are seen
in Figures A & B. What treatment do you suggest?
Dr.Wael Abboud (Foot and Ankle OB)

1. Observation
2. Medial sesamoidectomy
3. Cheilectomy and joint debridement
4. 1st MTP resection artrhoplasty (Keller procedure)
5. 1st MTP fusion

PREFERRED RESPONSE ▼ 3
CORRECT

This patient has hallux rigidus, or arthritis of the first metatarsal


phalangeal joint. She has moderate limitations in motion, and mild
joint space narrowing on radiographs. Therefore, she would be
classified as Type II, and the most appropriate treatment would be
cheilectomy and joint debridement.

Coughlin et al broke hallux rigidus down into grades zero thru 4,


based on range of motion, radiographic, and clinical findings. They
describe 97% good to excellent results with cheilectomy for grade
1 or 2 disease.

Mann et al describe their excellent results (inproved mobility and


decreased pain) in treating 25 patients with cheilectomy and
describe that cheilectomy offers a better functional treatment for
hallux rigidus.

Illustration A shows the different grades described by Coughlin et


al.
Dr.Wael Abboud (Foot and Ankle OB)

21-A 70-year-old sedentary female underwent a silastic


arthroplasty of the right 1st metatarsophalangeal joint 15
years ago. She now presents with pain, swelling and
erythema of the MTPJ. She is afebrile, bloodwork reveals
normal ESR, CRP and WBC, and her erythema resolves with
elevation. NSAIDs and activity modification have failed to
provide relief. What is the best option to treat her painful toe?

1. Steroid joint injection


2. Custom molded orthosis with recessed 1st metatarsal
molding
3. Irrigation and debridement and IV antibiotics
Dr.Wael Abboud (Foot and Ankle OB)

4. Revision of silastic implant and synovectomy


5. Removal of implant and synovectomy

PREFERRED RESPONSE ▼ 5
CORRECT

This patient with a silastic arthroplasty has late stage synovitis


from the implant. Normal laboratory values involving the white
blood cell count, crp and esr help rule out infection as well as the
clinical finding that the erythema mostly resolves with elevation.
Kitaoka et al showed that removal of the implant and synovectomy
of failed silastic arthroplasties of the 1st metatarsophalangeal joint
was an effective first line surgical treatment. Of the 10 patients
they were able to get follow up on, 7 did excellent, 1 did good, one
did fair, and 1 did poor. More recent studies and review articles
such as by Esway et al have shown that implant removal and
synovectomy can improve patient satsifaction after a failed
arthroplasty. The decision to treat a failed arthroplasty with
arthrodesis versus just implant removal depends on the functional
demands of the patient and alignment of the foot. Kitaoka et al
showed that lateral forefoot pressures increased following implant
removal, but there was no significant change in overall alignment.

22- A Morton's extension orthotic is used for which of the


following conditions?

1. Hallux rigidus
2. Hallux valgus
3. Hallux varus
4. Morton's neuroma
5. Spring ligament rupture

PREFERRED RESPONSE ▼ 1
CORRECT
Dr.Wael Abboud (Foot and Ankle OB)

Hallux rigidus refers to degenerative arthritis of the first


metatarsophalangeal (MTP) joint. Patients commonly present with
pain and stiffness at the first MTP joint. In the early stages, the
discomfort predominates at the dorsal aspect of the joint and
becomes more diffuse with the progression of the disease.
Walking worsens the pain, particularly during heel-rise and toe-off.
CAM boot, AFO, and hard soled shoes are not recommended first
line treatment options. The use of a Morton’s extension (see
illustration) will restrict motion. Other non surgical treatments
include modifications in shoewear, limitations in activity, and
injections with corticosteroid. Operative treatment includes joint
salvage and joint destructive procedures. These include
cheilectomies, osteotomies, arthrodesis, or arthroplasty.
The reference by Yee and Lau covers the current concepts behind
hallux rigidus, including the aforementioned staging and treatment
options.
Dr.Wael Abboud (Foot and Ankle OB)

23-A 45-year-old carpenter reports pain and restricted


dorsiflexion of the great toe. An x-ray reveals mild
osteoarthritis of the first metatarsophalangeal joint and a
prominent dorsal osteophyte. Orthotics have failed to provide
relief. What is the most appropriate intervention at this time?

1. cheilectomy
2. Lapidus procedure
3. first metatarsophalangeal arthrodesis
4. metatarsophalangeal resurfacing
5. chevron osteotomy

PREFERRED RESPONSE ▼ 1
CORRECT

Cheilectomy (1st metatarsal osteotomy) for treatment of hallux


rigidus will relieve dorsal impingement that is usually the source of
pain in patients with this condition. Cheilectomy is recommended
for mild to moderate deformity, ie. Grade1-2 and some 3’s.
Arthrodesis is treatment of choice following failed cheilectomy or
where advanced degenerative changes are present such as Grade
4 hallux rigidus and Grade 3 where <50% of the metatarsal head
cartilage remains at the time of surgery. Patient can also try
nonoperative options such as molded shoe inserts or rocker
bottom shoes.

24-A 54-year-old male carpenter is having pain and stiffness


in his great toe on the right foot that is exacerbated when he
kneels down on his right knee. The skin over the dorsal
surface of the 1st metatarsophalangeal joint reveals shoe-
wear irritation. He has attempted wearing a carbon fiber shank
in his shoe and a trial of meloxicam without relief of
symptoms. Radiographs are shown in Figures A and B. What
is the next most appropriate step in management?
Dr.Wael Abboud (Foot and Ankle OB)

1. Resection of dorsal osteophyte and 25% of the dorsal


aspect of the metatarsal head
2. Proximal phalanx medial closing wedge osteotomy
3. First metatarsophalangeal joint arthrodesis
4. Medial eminence removal and resection of base of
proximal phalanx
5. First tarsometatarsal joint arthrodesis and
metatarsophalangeal capsular release

PREFERRED RESPONSE ▼ 1
CORRECT

Hallux rigidus describes a degenerative arthritic process that leads


to functional limitation of motion in the first MTP joint. Patients
have tenderness over the dorsum of the first MTP joint and limited
dorsiflexion. Most patients with grades I and II changes can be
treated with cheilectomy with removal of the dorsal osteophyte and
25% of the dorsal aspect of the metatarsal head. The cheilectomy
allows dorsiflexion of the proximal phalanx on the metatarsal head.
The goal is to obtain 70° to 90° of dorsiflexion intraoperatively. A
patient whose main complaint is shoe wear irritation from the
prominence or pain with dorsiflexion is the best candidate for
cheilectomy.

Lau et al present Level 4 evidence of 19 patients with grade 2 and


11 patients with grade 3 hallux rigidus. The grade 2 arthritis
Dr.Wael Abboud (Foot and Ankle OB)

patients were treated with a cheilectomy whereas the grade 3 were


treated with interpositional arthroplasty. AOFAS scores were
higher in the cheilectomy group.

Feltham et al present Level 4 evidence of 67 patients treated with


cheilectomy with 91% being better than prior to surgery and patient
age did not affect clinical outcome.

A diagram of the above described cheilectomy is provided in


Illustration A.

25-What is the most common surgical complication of


resection of both the medial (tibial) and lateral (fibular)
hallucal sesamoids on the same foot for intractable
keratosis?

1. Hallux varus deformity


2. Neuroma
3. Toe cock-up deformity
4. Recurrence of keratosis
5. Hallux valgus deformity

PREFERRED RESPONSE ▼ 3
CORRECT
Dr.Wael Abboud (Foot and Ankle OB)

Excision of both sesamoids should always be avoided since it can


lead to a cock-up deformity, which is probably due to decreasing
the moment arm of the flexor hallucis brevis. This MTP extension
deformity leads to relative overpull of the flexors, causing IP joint
flexion. Excision of the tibial sesamoid alone can lead to hallux
valgus. Excision of the fibular sesamoid alone can lead to hallux
varus.
The referenced article by Richardson is an excellent review of
sesamoid pain and reviews the causes, diagnosis and treatment of
this pathological condition.

26-A 57-year-old man plays 45 holes of golf per week and has
foot pain during the toe-off phase of gait. He notes the foot
pain started 3 months ago after walking up a hill and falling
forward on some wet grass. Your exam shows skin callosities
dorsally at the 2nd PIP joint and plantarly at the 2nd MT head.
Radiographs show a hyperextension deformity of the 2nd
proximal phalanx in relation to the metatarsal. All of the
following are true about this patient's condition EXCEPT.

1. Symptomatic treatment initially includes extradepth


shoes, metatarsal pads, and well-padded liners
2. The MTP drawer test will likely show laxity at the MTP
joint in the dorsal-plantar plane
3. Repeated MTP dorsiflexion weakens the plantar
aponeurosis, plantar plate, and capsular stabilizers
4. Plantar callosities result from dorsal displacement of the
fatty cushion underneath the metatarsal head
5. Condition is a result of repetitive stresses causing
microfractures with subsequent compromised blood supply
to the metatarsal subchondral bone

PREFERRED RESPONSE ▼ 5
CORRECT
Dr.Wael Abboud (Foot and Ankle OB)

This patient's 2nd MTP hyperextension deformity is a result of a


plantar plate rupture. Answer 5 describes the disease process of
Freibergs Infraction and AVN of the subchondral bone of the
metatarsal. The JAAOS article by Mizel describes lesser MTP joint
conditions including MTP instability, subluxation, and dislocation.
Patients with plantar plate ruptures have pain with toe-off as MTP
dorsiflexion forces attenuate the plantar stabilizers and
subsequently lead to the hyperextension of the proximal phalanx.
A drawer test seen in Illustration A can demonstrate instability in
the dorsal-plantar plane with plantar plate incompetence. The
resulting MTP deformity leads to skin compromise at the PIP
dorsally secondary to abutting the shoe box and the plantar
metatarsal head due to dorsal migration of the plantar fatty
cushion. Initial treatment includes off-loading the areas of
increased pressure with metatarsal pads and relieving areas of
high skin pressure with indepth shoes. Surgical options include
resection of the metatarsal head to decompress the area and allow
the fatty cushion to resume its normal position.

27-A 56-year-old diabetic female presents with the painful


right toe deformity shown in Figure A. Physical exam reveals
MTP dorsiflexion, and flexion at the DIP and PIP joints. The
PIP and MTP joints are flexible, and she has failed
conservative treatment. Which of the following is the best
surgical treatment option for this patient?
Dr.Wael Abboud (Foot and Ankle OB)

1. Girdlestone-Taylor flexor-to-extensor tendon transfer


2. PIP arthrodesis
3. Complete MTP capsulotomy and resection arthroplasty
of the proximal phalanx with tendon release/lengthening
4. Complete MTP capsulotomy and resection arthroplasty
of the proximal phalanx with a Weil osteotomy
5. Isolated Weil osteotomy of the affected metatarsal

PREFERRED RESPONSE ▼ 1
CORRECT

Claw toe deformity occurs most commonly in patients with


diabetes, alcoholism, or an underlying neuromuscular condition.
The classic presentation is a dorsiflexed MTP joint, and
hyperflexion at the PIP and DIP joints. Surgical treatment is based
on whether or not the PIP joint is flexible or fixed. The Girdlestone-
Taylor procedure involves transferring the FDL to the extensor
surface of the affected toe, allowing the long toe flexors to behave
like intrinsic muscles producing active plantar flexion at the MP
joints and extension at IP joints. This is only effective in the
presence of a flexible deformity. PIP arthrodesis, MTP
capsulotomy, and PIP joint resection arthroplasty are only
indicated in varying degrees of a fixed claw toe deformity. A Weil
osteotomy can be added in these fixed cases for added correction,
or if multiple toes are involved. Illustration A shows a comparison
of the different lesser-toe deformities.
Dr.Wael Abboud (Foot and Ankle OB)

28-A 54-year-old female has struggled with 2nd metatarsalgia


that is not relieved with orthotics. She undergoes a metatarsal
shortening osteotomy using the technique demonstrated in
Figure A. Following surgery she complains that her 2nd toe
"floats" above the level of the remaining lesser toes. Which of
the following is the most likely cause?

FIGURES: A
Dr.Wael Abboud (Foot and Ankle OB)

1. Lack of appropriate post-operative immobilization


2. Technique of surgical osteotomy
3. Post-operative cerebrovascular event causing
neuromuscular imbalance
4. Inadvertent tethering of the extensor tendon during
surgical procedure
5. Surgical site infection

PREFERRED RESPONSE ▼ 2
Dr.Wael Abboud (Foot and Ankle OB)

CORRECT

Figure A demonstrates a Weil metatarsal shortening osteotomy


that results in mild depression of the joint surface in addition to
shortening of the metatarsal. This joint depression leads to a
dorsiflexion deformity of the MTP. To avoid this complication the
osteotomy should be made parallel to the plantar surface of the
foot or a wafer of bone should be resected to ensure that the joint
is not depressed as the metatarsal is shortened. These techniques
are demonstrated in Illustration A.

Trnka et al performed a cadaveric study to evaluate dorsiflexion


following Weil osteotomy. They found that plantar depression of
the MTP joint following the osteotomy altered the dynamic forces
of the interossei converting them from plantarflexors to
dorsiflexors.

In another article, Trnka et al reviewed 30 patients with


metatarsalgia who were treated with either metatarsal head
osteotomy (Weil) or metatarsal shaft osteotomy (Helal). Patients
treated with a Weil osteotomy had significantly higher satisfaction,
lower incidence of recurrent metatarsalgia, and fewer transfer
lesions.
Dr.Wael Abboud (Foot and Ankle OB)

29-A 57-year-old woman complains of pain and deformity of


the second toe that is limiting ambulation. Shoe
accomodations and NSAIDs have failed to provide relief. She
has a fixed flexion deformity of 40 degrees at the PIP joint, but
the MTP joint is not involved. The hallux is normal, but
painless PIP flexion contractures are present in the other
lesser toes. Which of the following is an indication for PIP
resection arthroplasty as opposed to soft-tissue balancing
and realignment?

1. Fixed deformity
2. Pain with shoe wear
3. Presence of hammertoe deformity in all lesser toes
Dr.Wael Abboud (Foot and Ankle OB)

4. Absence of metatarsophalangeal joint deformity


5. Absence of a concomitant hallux valgus deformity

PREFERRED RESPONSE ▼ 1
CORRECT

Hammertoe is defined as a flexion deformity of the proximal


interphalangeal joint. A diagram of the deformity is provided in
illustration A. There are many causes including neurogenic,
degenerative, and metabolic processes. Hammertoe can also be
secondary to another deformity in the foot such as hallux valgus or
curly toe. Consequently, there are numerous surgical procedures
available for treatment including soft tissue releases, tendon
transfers, resection arthroplasty, and arthrodesis. The presence of
fixed deformity precludes success with soft tissue procedures
alone. O'Kane et al review 100 cases of excisional arthroplasty for
second digit hammertoe. High rates of clinical satisfaction were
acheived with only 2 revision surgeries. Floating toe was the most
frequent complication (7). However, floating toe did not adversely
affect shoe wear in any patient experiencing this complication.
Coughlin et al review 63 cases of excisional arthroplasty for fixed
hammertoe. The resection arthroplasty was stabilized with K-wire
fixation that led to bony fusion in 81% and fibrous union in the
remaining 19%. Only minor complications occurred and 84% of
patients reported high clinical satisfaction.

30-Floating-toe deformity is the most common complication


of which of the following surgical procedures used to treat
central metatarsalgia?
Dr.Wael Abboud (Foot and Ankle OB)

1. Dorsal soft-tissue release with pin fixation


2. Silicone implant arthroplasty
3. MTP joint excisional arthroplasty
4. Metatarsal shaft osteotomy (Helal procedure)
5. Metatarsal neck osteotomy (Weil procedure)

PREFERRED RESPONSE ▼ 5
CORRECT

The Weil osteotomy is a joint preserving metatarsal head


osteotomy which is commonly used for the treatment of the
subluxed or dislocated joint lesser MTP joint associated with
central metatarsalgia. The most common complication associated
with the Weil osteotomy is the development of a floating-toe
deformity post-operatively.

Migues et al followed 70 Weil osteotomy patients for 18 months


post-operatively. They found a high occurence of floating-toe
deformity in the patients who underwent a concomitant PIP joint
arthrodesis. Although it was not associated with functional
impairment, they recommended that concurrent PIP arthrodesis be
avoided to reduce the occurrence of floating toes.

Highlander et al performed a literature review and reported on the


details of 1,131 Weil osteotomies. The most commonly reported
complication of the Weil osteotomy was floating toe, reported in
233 cases, with an overall occurrence of 36%.

Ilustration A shows the Weil osteotomy technique.


Dr.Wael Abboud (Foot and Ankle OB)

31-A 40-year-old man has metatarsalgia secondary to a


chronically dislocated 2nd metatarsalphalangeal(MTP) joint.
Nonoperative modalities including shoe modifications have
failed to improved his symptoms. When comparing osteotomy
B (Weil osteotomy) to osteotomy A (Helal osteotomy) as
shown in Figure A, all of the following are true EXCEPT:

FIGURES: A
Dr.Wael Abboud (Foot and Ankle OB)

1. Higher patient satisfaction rates


2. Lower incidence of recurrent metatarsalgia
3. Fewer transfer lesions
4. Higher percentage of radiographic reduction and
maintenance of the MTP joint reduction
5. Increased rate of malunion or pseudoarthrosis

PREFERRED RESPONSE ▼ 5
CORRECT

The goal of surgery for metatarsalgia and a dislocated MTP joint is


to improve pressure distribution within the forefoot following failure
of nonsurgical measures including shoe modifications. The Weil
procedure is an intra-articular osteotomy that achieves longitudinal
decompression through shortening and allows joint reduction. In a
Weil osteotomy the metatarsal (MT) is exposed and the direction
of shortening in the original Weil procedure runs mostly parallel to
the plantar aspect of the foot. The Weil osteotomy is fixed by
means of a screw running perpendicular to the osteotomy line. The
Helal osteotomy has no form of fixation and is made more
proximally. The article by Trnka et al compared 30 patients
receiving the 2 types of procedures and found that satisfaction
rates, AOFAS scores, malunion rates, pseudoarthrosis rates, and
MTP reduction rates were all better with the Weil osteotomy. The
article by Coughlin is a review article on lesser toe deformities.

32-A 70-year-old man complains of inability to wear normal


shoes on his left foot due to a second toe deformity.
Radiographs are shown in Figures A and B. He decides to
undergo surgical treatment. After intra-operative extensor
tendon lengthening and capsular release, the joint continues
to subluxate. What is the next step to correct the
deformity?
Dr.Wael Abboud (Foot and Ankle OB)

1. Flexor tendon resection


2. Proximal phalangeal cresenteric osteotomy
3. Metatarsophalangeal joint arthrodesis
Dr.Wael Abboud (Foot and Ankle OB)

4. Distraction osteogenesis of the metatarsal


5. Metatarsal shortening osteotomy

PREFERRED RESPONSE ▼ 5
CORRECT

With a mild/moderate deformity of the second


metatarsophalangeal joint, a release of capsule, tightening of
collateral ligaments, tendon lengthening, and flexor tendon
transfers can provide correction. Once the deformity turns
moderate to severe with subluxation or dislocation, the soft-tissue
release is often insufficient. Shortening osteotomies have been
recognized as effective joint decompression technique and
lengthening the adjacent soft-tissue structures. One technique is
the capital osteotomy of the metatarsal bone (such as the Helal or
Weil osteotomy). Once bone is shortened, the extensor tendon can
be lengthened and the flexor tendon can be transferred.

Trinka reported on 25 capital oblique osteotomies on 15 patients.


21 of the 25 MTP joint dislocations were successfully relocated
with an average shortening of 4.4mm. 12 of the 15 patients were
satisfied with the result. Complications included limited plantar
flexion and the recurrence (most frequent complication).

The reference by Coughlin is an Instructional Course Lecture


review article on lesser toe abnormalities and reviews the wide
variety of lesser toe pathology, including diagnosis and treatment.

Incorrect answers:
1: Flexor tendon resection would lead to a floppy toe.
2: Phalangeal base osteotomy would not lead to any change in
joint balance.
3: Joint arthrodesis would not be the next step in balancing the
unstable joint.
4: Distraction osteogenesis would lengthen the metatarsal, leading
to further instability/imbalance.
Dr.Wael Abboud (Foot and Ankle OB)

33-A diaphyseal fifth metatarsal osteotomy is the optimal


surgical treatment for which of the following patients who has
failed nonsurgical management?

1. 28-year-old woman with a Jones fracture nonunion


2. 50-year-old woman with a physiologic bow to the 5th
metatarsal and a lateral prominence of the 5th metatarsal
head
3. 54-year-old woman with a widened 4-5 intermetatarsal
angle and a lateral prominence of the 5th metatarsal head
4. 35-year-old woman with prominent lateral condyle of her
5th metatarsal head, without bowing of the metatarsal and
with normal 4-5 intermetatarsal angle
5. 60-year-old woman with rheumatoid arthritis and a
plantar plate rupture

PREFERRED RESPONSE ▼ 3
CORRECT

A bunionette, "tailor's bunion", is a lateral prominence of the fifth


metatarsal head, as seen in the photo in illustration A. Illustration B
shows a standard bunion and a bunionette on the same forefoot.
Usually a large 4-5 intermetatarsal (IM) angle and varus of the
metatarsophalangeal joint are associated with the tender
prominence, but not always. Conservative measures include wide
toe boxed shoes and pads. Surgical treatment is fairly rare and
only appropriate when nonoperative measures fail. A simple lateral
eminence resection alone is indicated when the 4-5 IM angle is
normal, but there is risk of recurrence and for overresection. A
distal chevron osteotomy is reliable for those with a lateral bow to
the fifth metatarsal, but a diaphyseal osteotomy of the 5th is
needed if the 4-5 IM angle is widened. Baumhauer et al. actually
state that they favor distal chevron osteotomies in most cases
because there is more reliable union in the more vascular distal
Dr.Wael Abboud (Foot and Ankle OB)

regions than in the diaphysis. However, Cohen et al. argue that


adequate correction can rarely be achieved without a diaphyseal
osteotomy when the 4-5 IM angle is increased. Osteotomies are
not generally used to treat Jones fracture nonunions. The patient
in scenario 2 would be best treated with a distal chevron
osteotomy, while the woman in scenario 4 could be treated with
lateral eminence resection alone. Rheumatoid patients with plantar
plate rupture can be treated with metatarsal head excision but not
with diaphyseal osteotomies.

34-A 44-year-old female has a Stage 2B acquired flat foot


deformity that does not improve over 6 months of
conservative management. She undergoes FDL tendon
transfer to the navicular, calcaneal osteotomy, and
tendoachilles lengthening. After this correction, it is thought
that she would benefit from a opening wedge first cuneiform
(Cotton) osteotomy. Which of the following best describes the
Cotton osteotomy?

1. Plantarflexion osteotomy to correct residual forefoot


varus
2. Medial opening wedge osteotomy to correct residual
forefoot abduction
3. Plantarflexion osteotomy to correct residual forefoot
valgus
4. Medial opening wedge osteotomy to correct residual
Dr.Wael Abboud (Foot and Ankle OB)

hindfoot abduction
5. Plantarflexion osteotomy to correct residual hindfoot
valgus

PREFERRED RESPONSE ▼ 1

Plantarflexion opening wedge medial cuneiform osteotomy is an


adjunctive procedure used to correct the forefoot varus (Illustration
A) component of a flatfoot deformity as described in the review
article by Tankson. In acquired flat foot deformity, the hindfoot falls
into valgus with compensatory varus developing in the forefoot.
After achieving surgical correction of the hindfoot to neutral,
residual forefoot varus is addressed to obtain optimal foot
reconstruction (Illustration B). Johnson et al performed a Level 4
study that showed that a Cotton osteotomy produced good results
and may offer advantages over 1st metatarsal-cuneiform fusion
including predictable union, preservation of first ray mobility, and
the ability to easily vary the amount of correction. Illustration C is a
drawing of the plantarflexion Cotton osteotomy.

35-A 53-year-old female has a 20 month history of left


hindfoot pain that has failed to respond to AFO bracing and
physical therapy. She has a unilateral planovalgus deformity,
Dr.Wael Abboud (Foot and Ankle OB)

shown in Figure A, which is flexible. She is unable to do a


single leg-heel rise. Which of the following surgical options is
most appropriate?

1. Triple arthrodesis
2. Isolated FDL transfer to the navicular
3. Dorsiflexion osteotomy of the 1st ray with peroneus
longus-to-brevis transfer
4. Lateralizing calcaneal osteotomy with FDL to navicular
transfer
5. Lateral column lengthening, medializing calcaneal
osteotomy, and FDL transfer to the navicular

PREFERRED RESPONSE ▼ 5
CORRECT

The patient is presenting with Stage II posterior tibial tendon


dysfunction (PTTD) as she is not able to do a single leg-heel rise
(Stage 1 can do a heel rise) and has a flexible deformity (Stage 3
is not flexible). First line of treatment, regardless of stage, is non-
operative. Operative management of Stage II disease is
convtroversial, but there is a growing interest in combining bone
procedures with tendon transfer to replace the incompetent PTT
which relieves pain and corrects the underlying deformity.
Alvarez et al assessed the efficacy of AFO bracing and exercise in
the treatment of 47 patients with Stage I or II PTTD. At 4 months,
Dr.Wael Abboud (Foot and Ankle OB)

83% of the patients had successful subjective and functional


outcomes, and 89% were satisfied.
Imhauser et al performed a cadaveric study to compare the
efficacy of in-shoe orthoses and ankle braces in stabilizing the
hindfoot and arch in PTTD. They concluded that treatment of
flatfoot deformity should at least include use of in-shoe orthoses to
partially restore the arch and stabilize the hindfoot.

36-The lower limb orthosis shown in Figure A is the most


effective method for nonsurgical management in which of the
following conditions?

1. Hallux valgus
2. Midfoot arthritis
3. Hallux rigidus
4. Diabetic foot neuropathy
5. Acquired flexible flatfoot deformity

PREFERRED RESPONSE ▼ 5
CORRECT

The orthosis in Figure A is an Ankle-foot orthosis (AFO) and is


most appropriately used in the setting of a flexible flatfoot
deformity. Alvarez et al studied a cohort of patients undergoing a
physcial therapy program and articulating AFO use and found an
89% satisfaction rate. Chao et al found 67% good to excellent
results with use of a molded AFO or UCBL orthosis (University of
California Biomechanics Laboratory orthosis is constructed of
plastic and is fabricated over a cast of the foot held in maximal
manual correction shown in Illustration A). Augustin et al found
Dr.Wael Abboud (Foot and Ankle OB)

good results with an Arizona AFO (Arizona brace combines the


UCBL orthosis with a laced ankle support shown in Illustration B).
Hallux valgus can be treated with a wide shoe box and cushions
over the prominence. Midfoot arthritis can be treated with a rigid
sole and rocker bottom. Hallux rigidus is best addressed
nonsurgically with a rigid forefoot orthosis that limits 1st MTP
dorsiflexion. Diabetic foot neuropathy is best addressed with
pressure dissipating insoles and accomodative laced shoes.

37-A 56-year-old woman comes to your office with foot pain


after a 9 month trial of orthotics. Your examination reveals the
hindfoot is in valgus, the arch is depressed, and the forefoot
is abducted when the foot is viewed posteriorly. She is unable
to perform a single heel rise on the affected side. The hindfoot
is flexible and there is an equinus contracture. What
combination of surgical interventions is most appropriate

1. Tenosynovectomy followed by UCBL orthotic use


2. Dwyer closing wedge calcaneal osteotomy, 1st
metatarsal closing wedge osteotomy, and plantar fasica
release
3. Medial calcaneal displacement osteotomy, FDL tendon
transfer, and tendoachilles lengthening
4. Arthrodesis of the subtalar, talonavicular, and
calcaneocuboid
5. Lateral calcaneal displacement osteotomy, FDL tendon
transfer, and tendoachilles lengthening
Dr.Wael Abboud (Foot and Ankle OB)

PREFERRED RESPONSE ▼ 3
CORRECT

This patient presents with a Stage 2B acquired flatfoot deformity.


The review articles by Myerson and Pinney discuss the
classification and management of adult acquired flatfoot deformity.
Stage 1 presents with medial ankle pain due to posterior tibial
tendon synovitis. Stage 2 presents with a hindfoot in valgus and
inability to perform a single limb heel rise. Stage 2A has a normal
forefoot whereas Stage 2B has an abducted forefoot, exposing
“too many toes” when the foot is viewed posteriorly. Stage 3
occurs over time as the hindfoot becomes rigid in a valgus position
and Stage 4 develops as the deltoid ligament becomes
incompetent and the talus tilts into valgus. Tenosynovectomy can
be used in Stage 1 disease that fails conservative management.
Answer 2 is used as a treatment for cavovarus feet. Answer 4 is a
treatment option for Stage 3 disease. Answer 5 is not an option as
a lateral displacement calcaneal osteotomy would exacerbate the
patient's hindfoot valgus.

38-A 70-year-old female complains of progressive pain of the


medial ankle and foot over the past 10 years. Orthotics no
longer provide relief of her pain. The hindfoot deformity is
unable to be passively corrected on physical exam. Figure A
is a posterior view of the patient's foot upon standing and a
current radiograph is provided in Figure B. Which of the
following is the best treatment option?
Dr.Wael Abboud (Foot and Ankle OB)

1. Posterior tibialis tendon debridement


2. FDL transfer to navicular and calcaneal slide osteotomy
3. FDL transfer to navicular, calcaneal slide osteotomy,
and lateral column lengthening through the cuboid
4. Talocalcaneal arthrodesis
5. Triple arthrodesis

PREFERRED RESPONSE ▼ 5
CORRECT

Image demonstrates the "too many toes" sign consistent with


posterior tibialis tendon dysfunction. Failure of the hindfoot to
correct during physical exam renders this a fixed rather than
flexible deformity (Stage III) warranting triple arthrodesis. Flexible
deformity (Stage II) may be surgically treated with calcaneal
osteotomy, FDL transfer, and lateral column lengthening. The
review article by Deland and ICL by Myerson review the
evaluation, diagnosis, and management of adult-acquired flatfoot
deformity.
Dr.Wael Abboud (Foot and Ankle OB)

39-What is the preferred surgical treatment for painful


acquired flatfoot deformity with stage III posterior tibial
tendon insufficiency?

1. FDL transfer to the navicular, medial displacement


calcaneal osteotomy, and tendoachilles lengthening
2. Pantalar arthrodesis
3. FDL transfer to the navicular with lateral column
lengthening through the anterior calcaneus
4. Posterior tibial tendon debridement and tenodesis to
FDL
5. Arthrodesis of calcaneocuboid, talonavicular, and
subtalar joints

PREFERRED RESPONSE ▼ 5
CORRECT

Stage III posterior tendon insufficiency is characterized by a fixed,


rather than flexible deformity, with associated subtalar and midfoot
arthritis which warrants triple arthrodesis (fusion of
calcaneocuboid, talonavicular, and subtalar joints). Flexible
deformity (Stage II) may be surgically treated with calcaneal
osteotomy, FDL transfer, and lateral column lengthening. In stage
IV the talotibial ankle joint is also arthritic, or there is talar tilt, and a
pantalar fusion is warranted. The ICL by Myerson and Current
Concept Review by Pinney and Lin review the evaluation,
diagnosis, and management of adult-acquired flatfoot deformity.

40-An obese 65-year-old woman has a chronic painful flatfoot


with a rigid valgus hindfoot deformity. Radiographs reveal
subtalar joint degenerative changes but no signs of ankle
joint degenerative changes or abnormal talar tilt. She is
unable to single-leg toe raise and has a "too many toes" sign.
What stage of posterior tibial tendon dysfunction is she best
classified as?
Dr.Wael Abboud (Foot and Ankle OB)

1. V
2. IV
3. III
4. II
5. I

PREFERRED RESPONSE ▼ 3
CORRECT

Acquired pes planus deformity in adults is most often caused by


posterior tibial tendon (PTT) insufficiency. Classic cases occur in
overweight, middle aged patients. Physical exam findings include
too many toes sign. In the referenced articles by Myerson and
Pinney, 4 stages of the pathology are listed. They are: Stage 1:
tenosynovitis of the PTT and maintenance of the ability to perform
a single-leg toe raise; Stage 2: flexible valgus hindfoot deformity
and unable to perform single leg toe raise; Stage 3: fixed valgus
hindfoot deformity; Stage 4: Stage III changes with the addition of
ankle degenerative changes and possible deltoid ligament
complex insufficiency.

41-A 46-year-old obese female presents with foot pain and the
radiographs shown in Figures A and B. Which of the following
physical findings will most likely be present?
Dr.Wael Abboud (Foot and Ankle OB)

1. Achilles tendon contracture


2. Hallux varus
3. Forefoot adduction
4. Hindfoot varus
5. Clawing of the toes

PREFERRED RESPONSE ▼ 1
CORRECT

The radiographs shown in Figures A and B demonstrate adult-


acquired flatfoot deformity secondary to posterior tibial tendon
dysfunction. Note the uncovering of the talar head on the AP
radiograph, and decreased distance between the medial cuneiform
and floor on the lateral radiograph indicating collapse of the medial
longitudinal arch. In patients with this deformity, the medial
longitudinal arch collapses, the subtalar joint everts and the heel
assumes a valgus position. As a result of the hindfoot valgus, the
Achilles tendon is held lateral to axis of rotation of subtalar joint
and begins to act as an evertor of the calcaneus. Untreated
deformity can lead to a fixed equinus deformity of the hindfoot and
an Achilles tendon contracture. Myerson reviews the etiology,
classification, evaluation, and treatment of posterior tibial tendon
dysfunction in his ICL. Pinney et al point out in their CCR that
when testing ankle dorsiflexion, the hindfoot must be held in a
neutral position to allow the true extent of equinus contracture
become apparent.

42-A healthy 42-year-old male has a 2-year history of


worsening hindfoot pain that is refractory to therapy and
orthotics. Physical exam reveals a flexible planovalgus foot
with an equinus contracture. He is unable to perform a single
limb heel rise on the affected side. In addition to a flexor
digitorum longus tendon transfer to the navicular, which of
the following operative procedures is indicated?
Dr.Wael Abboud (Foot and Ankle OB)

1. Gastrocnemius lengthening only


2. Triple artthrodesis and gastrocnemius lengthening
3. Subtalar arthrodesis and gastrocnemius lengthening
4. Lateralizing calcaneal osteotomy, medial column
lengthening, and gastrocnemius lengthening
5. Medializing calcaneal osteotomy, lateral column
lengthening, and gastrocnemius lengthening

PREFERRED RESPONSE ▼ 5
CORRECT

This patient has stage II posterior tibial tendon insufficiency, a


flexible pes planovalgus deformity and inability to perform a single
limb heel rise. Operative treatment includes gastrocnemius
lengthening to improve dorsiflexion, FDL transfer to the navicular
to reproduce posterior tibial tendon function, lateral column
lengthening, and medializing calcaneal osteotomy to correct bony
deformity. Moseir-LaClair et al report 5 year follow-up of 26
patients with Stage II treated with this constellation of procedures.
They reported a mean postoperative ankle-hindfoot score of 90
and no nonunions. Four patients (14%) displayed radiographic
signs of calcaneocuboid arthritis at follow-up with only one that
was symptomatic and required calcaneocuboid joint fusion.
Arthrodesis is not indicated for flexible deformities. Myerson et al
reviewed the radiographic follow-up of 18 patients who underwent
FDL transfer and medializing calcaneal osteotomy for flexible pes
planovalgus. Radiographs demonstrated reduction of the
magnitude of deformity, and the authors hypothesized that the
bony reconstruction optimizes the dynamic forces of the FDL.
Dr.Wael Abboud (Foot and Ankle OB)

43-A 55-year-old woman presents with a planovalgus


deformity of her foot. She is unable to perform a single-limb
heel rise and has tenderness and swelling behind the medial
malleolus. Her hindfoot valgus is passively correctable and
she has failed a trial of orthotics. What is the most
appropriate treatment?

1. posterior tibial tendon debridement


2. medial displacement calcaneal osteotomy and posterior
tibial augmentation with flexor digitorum longus tendon
transfer
3. triple arthrodesis
4. ankle fusion
5. Lapidus procedure

PREFERRED RESPONSE ▼ 2
CORRECT

Stage I disease involves mild weakness of the posterior tibial


tendon (seen best with heel rise) and a mobile and well aligned
hindfoot. Stage I posterior tibial tendonitis (PTT) insufficiency is
treated nonoperatively. Stage II PTT is characterized by flexible
flatfoot, stage III is a rigid flatfoot, and stage IV has a rigid hindfoot,
valgus angulation of the talus and tibiotalar arthritis. Medial
displacement calcaneal osteotomy and flexor digitorum longus
transfer to the navicular is a classic surgical treatment for flatfoot
deformity of stage II. Stages III and IV require some type of fusion.
Myerson et al retrospectively looked at their results for Stage II
PTT with a FDL transfer and medial calcaneal osteotomy and
found excellent clinical scores at follow up.

44-An 40-year-old male with a progressive planovalgus foot


deformity secondary to posterior tibial tendon insufficiency
(PTTI) has failed nonoperative treatment. What feature must
be evaluated for that is commonly seen in patients with
advanced PTTI and should be addressed at time of surgery?

1. Plantar fascittis
2. Equinus contracture
Dr.Wael Abboud (Foot and Ankle OB)

3. Claw toes
4. Hallux varus
5. Hallux valgus

PREFERRED RESPONSE ▼ 2
CORRECT

With adult aquired pes planus it is important to look at how tight


the gastrocnemius-soleus complex is, so that can be addressed if
surgical intervention is necessary. Posterior tibial tendon
dysfunction is a commonly tested question. Acquired pes planus
deformity in adults is generally caused by posterior tibial tendon
insufficiency, a vital hindfoot inverter. As the dysfunction
progresses, the deformity that ensues are the following: hindfoot
valgus, midtarsal abduction and forefoot supination. In regards to
this question, with valgus deformity of the hindfoot, the Achilles
tendon insertion remains lateral to the rotation of the subtalar joint.
Loss of the longitudinal arch results in a fixed equinus deformity of
the hindfoot and contracture of the Achilles tendon. Hallux valgus
has not been shown to have any increased rate in PTTI compared
to the general population.

45-When harvesting flexor digitorum longus (FDL) for a


tendon transfer for stage II posterior tibialis tendon
dysfunction, what anatomic structure crosses immediately
deep (dorsal) to it in the midfoot region?

1. achilles tendon
2. flexor hallucis brevis (FHB)
3. adductor hallucis
4. flexor hallucis longus (FHL)
5. abductor hallucis

PREFERRED RESPONSE ▼ 4
CORRECT

In the midfoot FHL runs immediately deep (dorsal) to FDL.


When the FDL and FHL are in the ankle, FDL is medial to the FHL
("Tom, Dick and Harry"). However they cross each other at the
Dr.Wael Abboud (Foot and Ankle OB)

Knot of Henry, where the FHL goes anterior to the FDL, and then
tracks immediately deep (dorsal) to the FDL in the midfoot.

Illustration A shows the relative position of FDL to FHL at the ankle


and plantar aspect of the foot.
Illustration B shows the relative position of FHL to FDL from a
plantar view of the foot
Illustration V reviews the anatomy of the plantar surface of the foot,
and at 56 seconds demonstrates the relative position of FHL to
FDL.

Wrong Answers:
Answer 1: The achilles tendon should not be seen in a harvest
exposure.
Answer 2: The FHB is anterior and slightly lateral to the FHL.
Answer 3: Adductor hallucis crosses too distal for harvest.
Answer 5: Abductor hallucis never crosses the FDL.
Dr.Wael Abboud (Foot and Ankle OB)
Dr.Wael Abboud (Foot and Ankle OB)
Dr.Wael Abboud (Foot and Ankle OB)

46-A 45-year-old male presents with complaints that his left


foot "slaps" on the floor when he tries to ambulate. He reports
a remote history of playing rugby 7 months ago when an
opposing player fell on his plantarflexed left ankle. He denies
constitutional symptoms. On physical examination he has
weak dorsiflexion and increased fatigue with walking. He is
able to achieve 15 degrees of passive dorsiflexion with the
knee in full extension and 20 degrees of dorsiflexion with his
knee in 90 degrees of flexion. A sagittal T2 MRI is shown in
Figure A and axial MRI images are shown in Figures B and C.
Dr.Wael Abboud (Foot and Ankle OB)

Which of the following is the MOST appropriate next step in


management?

1. Surgical reconstruction with posterior tibial tendon


transfer and gastrocnemius recession
2. MRI of the proximal tibiofibular joint for evaluation of
ganglion cyst and EMG of the peroneal nerve
3. Primary surgical repair with gastrocnemius recession
4. Chest CT, skeletal survey, hematology profile, and
referral to an orthopaedic oncologist for biopsy of the mass
5. Surgical reconstruction with plantaris tendon
interposition augmentation

PREFERRED RESPONSE ▼ 5
CORRECT
Dr.Wael Abboud (Foot and Ankle OB)

This patient's history, examination, and images are consistent with


chronic anterior tibialis rupture. This injury most often occurs in
middle-aged patients following an eccentric loading of a
degenerated tibialis anterior tendon against a plantar flexed foot.
The classic triad for anterior tibialis ruptures include: (1) a
pseudotumor at the anterior part of the ankle that corresponded
with the ruptured tendon end, (2) loss of the normal contour of the
tendon, and (3) weak dorsiflexion of the ankle accompanied by
hyperextension of all of the toes can help to make the diagnosis.

Sammarco et al present a Level 4 review of 19 patients with


traumatic or atraumatic ruptures that were surgically treated in an
early (<6 weeks) or late (>7 weeks) manner. Patients who were
managed surgically both early and late had improvements in
dorsiflexion strength and gait pattern and had a significant
improvement in the AOFAS hindfoot score as compared with the
preoperative value. The authors advocate repair or reconstruction
of the tendon to restore dorsiflexion and inversion of the ankle in
order to approximate a normal gait pattern and theoretically to
avoid the late development of foot deformity

Ouzounian et al present a Level 4 review of 12 patients with


chronic anterior tibialis ruptures. Two types of ruptures were
identified on the basis of clinical presentation: (1) atraumatic
ruptures, which occurred in low-demand older patients who
presented late with minimal dysfunction and (2) traumatic ruptures,
which occurred in higher-demand younger patients who presented
earlier with more disability. The authors concluded (1) that patients
with traumatic ruptures, regardless of the time of presentation,
demonstrated better function after operative intervention and (2)
that patients with atraumatic ruptures who present early should be
managed surgically, whereas those with delayed presentation
could be managed with bracing.

Illustration A demonstrates a loss of normal contour of the anterior


tibialis tendon of the right ankle associated with a rupture of the
anterior tibialis tendon.
Dr.Wael Abboud (Foot and Ankle OB)

47-A 58-year-old golfer fell stepping into a sand trap and


ruptured his achilles tendon one year ago. He initially chose
non-operative treatment, but became unsatisfied with a tender
fullness behind his ankle and ankle weakness noticeable
during his tee shots. At the time of surgery, a large
disorganized fibrous mass is found at the site of rupture.
Following extensive debridement there is a 5 cm gap between
viable tissue ends. Which of the following surgical techniques
provides the greatest likelihood of a successful clinical
outcome?

QID: 3124

1. Gastocnemius turndown repair augmented with transfer


of the posterior tibial tendon
2. Gastocnemius turndown repair augmented with transfer
of the extensor digitorum longus
3. Gastocnemius turndown repair augmented with transfer
of the flexor hallucis longus
4. Reconstruction with hamstring autograft
Dr.Wael Abboud (Foot and Ankle OB)

5. Primary repair with the foot in maximal plantarflexion


followed by a gradual stretching program

PREFERRED RESPONSE ▼ 3
CORRECT

Tendon loss is a complication associated with secondary ruptures


of a repaired achilles tendon and chronic achilles tendon ruptures.
Gastrocnemius turndown utilizes a slip of the central third of the
gastrocnemius tendon to bridge the gap. Flexor hallicus longus
(FHL) is the preferred tendon transfer to augment tissue loss due
to its proximity and vascularity. Answer choice 5 is not a prudent
option given the risk of equinus contracture and recurrent rupture.
Wapner et al conducted a case review of 7 patients who
underwent FHL augmentation for chronic achilles tendon rupture.
Results included no surgical complications, a small but functionally
insignificant decrease in ankle and great toe range of motion, and
clinical satisfaction of all 7 patients. Chiodo et al summarize the
AAOS clinical guidelines for evaluation and treatment of acute
achilles tendon ruptures predicated upon an extensive review of
the literature.

48-What is the greatest advantage of surgical repair of an


acute Achilles tendon rupture with early range of motion
compared to non-operative treatment with immobilization in a
short-leg cast for 6 weeks?

1. Lower rate of infection


2. Higher rate of normal skin sensation
3. Better skin cosmesis
Dr.Wael Abboud (Foot and Ankle OB)

4. Lower rate of dehiscence


5. Lower rate of re-rupture

PREFERRED RESPONSE ▼ 5
CORRECT

Operative treatment with early range of motion has a lower rate of


rerupture compared to immobilization in a short leg cast for 6
weeks.

However, there is recent evidence that an aggressive nonoperative


rehabilitation protocol can have equivalent rates of rerupture
compared to operative treatment. Wilits et al performed a Level 1
study with an aggressive rehabiliation protocol with weightbearing
and range of motion starting at the 2 week interval(shown in
Illustration A) for achilles tendon ruptures. They found that there
was no clinically important difference between groups with regard
to strength, range of motion, calf circumference, or Leppilahti
score. There were twice as many complications in the operative
group than the nonoperative group with the main difference being
the greater number of soft-tissue-related complications..

In a pooled statistical analysis of 6 randomized studies by


Bhandari et al, surgical repair significantly reduced the risk of
rerupture when compared with nonsurgical treatment. However, it
increases the risk of infection and skin complications.

khan et al found that for postoperative immobilization, functional


bracing has been shown to have a significantly lower rate of
complications compared with casting, particularly with regard to
adhesion formation. Other complications with casting post-
operatively include disturbed sensibility, keloid hypertrophic
scarring, and infection.
Dr.Wael Abboud (Foot and Ankle OB)

49-A 38-year-old patient has an acute Achilles tendon rupture.


He is active in sports and is deciding between operative and
nonoperative treatments. Which of the following statements
applies to patients undergoing nonoperative treatment?

1. They have lower patient satisfaction scores


2. They are less likely to return to sport
3. Their ultimate strength is decreased
4. They have a higher risk for rerupture
5. They have a higher risk of skin problems

PREFERRED RESPONSE ▼ 4
CORRECT

This is a classic tested topic for which the answer has been a
higher risk of rerupture in the nonoperative group. This may no
longer be true. Weber et al retrospectively compared the results of
nonoperative and operative management of Achilles tendon
ruptures. They found that patient satisfaction, return to sports, and
ultimate strength was the same for both groups. The complication
Dr.Wael Abboud (Foot and Ankle OB)

rate was similar except for reruptures, with more occurring in the
nonoperative treated group versus the operatively treated group.
Khan et al conducted a meta-analysis of randomized controlled
trials and they found that open operative treatment of acute
Achilles tendon ruptures significantly reduces the risk of rerupture
compared with nonoperative treatment, but operative treatment is
associated with a significantly higher risk of other complications. A
more recent randomized study by Willits et al showed no statistical
difference in rerupture rates, which has created new controversy
on this subject.

50-Which factor increases the chance of wound complications


after Achilles tendon repair?

QID: 281

1. increased body mass index


2. immediate surgery
3. male gender
4. age over 40 years old
5. tobacco use

PREFERRED RESPONSE ▼ 5
CORRECT

Bruggeman et al anlayzed risk factors for wound complications


after Achilles tendon repair. They found that tobacco use, steroid
use, and female sex were significant risk factors for development
of wound complications. Over 40% of those patients who had one
or more of the following risk factors: diabetes, tobacco use, or
steroid use had a complication, compared with 6% for those
without risk factors present. Timing of surgery was not examined.

51-A 41-year-old female elects non-operative managment of


her acute achilles tendon rupture. What is the disadvantage of
casting over functional bracing?

QID: 1241
Dr.Wael Abboud (Foot and Ankle OB)

1. increased wound complications


2. decreased ankle range of motion
3. decreased strength
4. increased re-rupture rate
5. worse functional outcomes

PREFERRED RESPONSE ▼ 2
CORRECT

Saleh et al followed forty patients with acute complete rupture of


the achilles tendon who were allocated to treatment groups using
either cast immobilization for eight weeks or cast immobilization for
three weeks, followed by functional bracing. Patients treated with
functional bracing had improved ankle range of motion compared
to the immobilization group (p < 0.001). In addition, patients
treated with functional bracing were able to return to normal
activities sooner. Recovery of the power of plantar flexion was
similar in the two treatment groups, and no patient had excessive
lengthening of the tendon. One re-rupture occurred in each group.

52-A 48-year-old male complains of 5 years of heel pain while


running. Initially the pain was relieved with achilles tendon
stretching, orthotics, and open-backed shoe wear. Over the
past year these modalities are no longer helpful and he is
beginning to have pain with walking. Clinical photograph and
radiograph are provided in figures A and B. Which of the
following treatment options is the best choice to relieve pain
and improve function?

FIGURES: A B
Dr.Wael Abboud (Foot and Ankle OB)

1. Arizona gauntlet brace


2. Steroid injection
Dr.Wael Abboud (Foot and Ankle OB)

3. Achilles tendon debridement


4. Achilles tendon debridement, calcaneal exostectomy,
and FHL transfer
5. Ankle arthrodesis

PREFERRED RESPONSE ▼ 4
CORRECT

Clinical photograph and radiograph demonstrate Haglund's


deformity and calcifications consistent with insertional achilles
tendonopathy. Failure of conservative management and loss of
function are indications for surgical management. Given the large
Haglund's deformity on radiograph, calcaneal exostectomy is
preferable to tendon debridement alone. McGarvey reviewed the
clinical results of 22 insertional achilles tendonopathy treated
surgically finding a clinical satisfaction rate of 82%. Hartog reports
on 29 cases of FHL augmentation of chronic achilles tendonosis
finding excellent or good results in 26 of 29 and no report of
functional deficit or deformity of the hallux. Kolodziej conducted a
cadaveric study to evaluate the integrity of the insertion of the
achilles tendon. The greatest margin of safety was found to be
offered by a superior to inferior resection (better than medial/lateral
and oblique) and that as much as 50% of the tendon could be
resected without sacrificing significant strength to failure.

53-A 35-year-old man injured his ankle while playing soccer


two years ago. Ever since he has had persistent right ankle
pain that has failed to improve with nonoperative modalities
including physical therapy. A video of his right ankle is found
below. Radiographs are shown in Figures A through C. What
is the most appropriate next step in management?

FIGURES: V A B C
Dr.Wael Abboud (Foot and Ankle OB)
Dr.Wael Abboud (Foot and Ankle OB)
Dr.Wael Abboud (Foot and Ankle OB)
Dr.Wael Abboud (Foot and Ankle OB)

1. Physical therapy directed at proprioception and


strengthening
2. Fibular shortening osteotomy
3. Surgical repair of the anterior talofibular ligament (ATFL)
4. Surgical repair of the calcaneofibular ligament (CFL)
5. Fibular groove deepening and superior peroneal
retinaculum repair

PREFERRED RESPONSE ▼ 5
CORRECT

The patient in this scenario has chronic peroneal subluxation.


Radiographs are often normal in this condition. On physical exam,
a consistent finding is subluxation of the tendons with resisted
dorsiflexion and eversion of the ankle. This is most likely
secondary to a disruption of the superior peroneal retinaculum
Dr.Wael Abboud (Foot and Ankle OB)

(SPR). After a trial of non-operative management, surgical options


include repair of the SPR, with or without fibular groove deepening.

Philbin et al reviewed peroneal tendon injuries. As 40% lead to


chronic lateral ankle pain, they emphasized an accurate physical
exam. They concluded that low-demand patients do well with a
nonsurgical approach while high-demand patients may benefit
from surgery.

Heckman et al reviewed operative techniques for peroneal


disorders. They determined that operative repair consists of
surgical repair of the (SPR) with or without fibular groove
deepening for subluxation. Primary repair of the tendon is
indicated for tears <50% while tenodesis is indicated for tears
>50%.

The video depicts the clinical appearance of the peroneal tendons


subluxing with ankle dorsiflexion.

Incorrect answers:
1. He has already completed a trial of non-operative management.
2. A fibular shortening osteotomy is not the treatment of this
condition.
3. His exam is not consistent with an ATFL injury.
4. His exam is not consistent with a CFL injury.

54-A 24-year-old female sprains her ankle playing tennis. After


3 months of bracing, physical therapy, and NSAID treatment
she continues to complain of pain and a popping sensation
over the lateral ankle. Physical exam is notable for tenderness
over the lateral malleolus. Figure A shows the ankle at rest.
Figure B shows the ankle during active eversion. Which of the
following structures has been injured?

FIGURES: A B
Dr.Wael Abboud (Foot and Ankle OB)
Dr.Wael Abboud (Foot and Ankle OB)

1. Peroneus brevis
2. Inferior peroneal retinaculum
3. Superior peroneal retinaculum
4. Anterior talofibular ligament
5. Lateral process of the talus

PREFERRED RESPONSE ▼ 3
CORRECT

The clinical photographs demonstrate subluxation of the peroneal


tendons with eversion of the foot. This painful phenomenon occurs
following rupture of the superior peroneal retinaculum, which
keeps the peroneal tendons contained within the retromalleolar
groove of the fibula. Treatment initially consists of conservative
modalities including physical therapy. Failure of nonoperative
treatment warrants surgical repair of the disrupted retinaculum and
deepening of the groove if needed. An anatomic diagram of the
peroneal tendons running through the superior and inferior
retinaculum is provided in illustration A. Philbin et al reviews the
anatomy, evaluation, and treatment of peroneal tendon injuries.
Heckman et al reviews the different options for peroneal tendon
injuries.
Dr.Wael Abboud (Foot and Ankle OB)

55-In the retromalleolar groove, as shown in Figure A, what is


the relationship of the peroneus brevis tendon to the
peroneus longus tendon?

FIGURES: A
Dr.Wael Abboud (Foot and Ankle OB)

1. Anterior
2. Posterior
3. Medial
4. The peroneus longus tendon is not in the groove
5. The peroneus brevis tendon is not in the groove

PREFERRED RESPONSE ▼ 1
CORRECT

At the level of the ankle the peroneal tendons are contained in the
retromalleolar sulcus on the fibula. The sulcus is deepened by a
fibrocartilaginous rim and covered by the superior peroneal
retinaculum. In the retromalleolar sulcus at the level of the ankle
joint the peroneus brevis tendon lies anterior to the peroneus
longus tendon (see Illustration A and B). Selmani et al provide a
Dr.Wael Abboud (Foot and Ankle OB)

comprehensive review of peroneal tendon pathology and its


treatment. The three broad categories of peroneal tendon
pathology are peroneal tendinitis, peroneal subluxation, and
peroneal tendon tears. Often these issues are interrelated and can
even be progressive. Philbin et al also provide a review of the topic
discussing that the peroneal tendons are the primary evertors of
the foot and function as lateral ankle stabilizer. In their treatment
section they mention that lower demand patients may be
candidates for nonoperative management, whereas high demand
patients will probably benefit from surgical treatment.
Dr.Wael Abboud (Foot and Ankle OB)

56-A 17-year-old tennis player sustained an ankle sprain 4


weeks ago and now complains of painful popping behind the
lateral malleolus. What physical exam will reproduce his
symptoms?

QID: 702
Dr.Wael Abboud (Foot and Ankle OB)

1. External rotation stress test


2. Resisted inversion
3. Resisted dorsiflexion
4. Resisted eversion
5. Resisted plantar flexion

PREFERRED RESPONSE ▼ 4
CORRECT

The only structures behind lateral malleolus are peroneal tendons.


The peroneal tendons are a common source of pathlogy in
refractory ankle sprains. This patient is complaining of subluxing
peroneal tendon(s) as a result of a torn superior retinaculum. This
can be tested with resisted eversion as the peroneal tendons evert
the ankle. Maffulli et al concluded that an anatomic repair of the
retinaculum lead to good results in higher demand patients.

57-Which of the following mechanisms of injury to the ankle is


most likely to result in peroneal tendon instability?

QID: 1144

1. Plantarflexion and eversion


2. Dorsiflexion and eversion
3. Neutral ankle flexion and inversion
4. Dorsiflexion and inversion
5. Plantarflexion and inversion

PREFERRED RESPONSE ▼ 4
CORRECT

Peroneal tendon instability occurs during an inversion injury to a


dorsiflexed ankle with rapid reflexive contraction of the peroneus
longus and peroneus brevis tendons. Patients often hear a pop” or
feel a snapping sensation, followed by pain and swelling. The
Dr.Wael Abboud (Foot and Ankle OB)

peroneals have a vascular watershed region just posterior to the


fibula and are prone to longitudinal tears. Radiographs often show
an avulsion fracture of the distal fibula (rim fracture) at the insertion
of the superior peroneal retinaculum.Chronic instability of the
peroneal tendons can be best demonstrated by positioning the
ankle in dorsiflexion and resisting eversion. Treatment for acute
injuries involves cast immobilization to allow the superior peroneal
retinaculum to heal. Injuries recalcitrant to conservative
management or high level athletes may benefit from superficial
peroneal retinaculum repair and fibular groove-deepening
procedures.

58-A 30-year-old ballet dancer complains of snapping and


pain behind the medial malleolus, as well as triggering of her
great toe. MRI scan is shown in figure A. What antatomic
structure is being impinged at the level of the posterior ankle
joint?

FIGURES: A
Dr.Wael Abboud (Foot and Ankle OB)

1. posterior tibial tendon


2. extensor hallucis longus tendon
3. flexor hallucis longus tendon
4. tibial nerve
5. anterior tibial tendon

PREFERRED RESPONSE ▼ 3
CORRECT

The clinical presentation and MRI are consistent with FHL


tendonitis. More specifically, the MRI shows prominent
tenosynovitis of the sheath of the flexor hallucis longus (FHL)
tendon as it wraps around the posteromedial ankle (see illustration
below). Impingement of the flexor hallucis longus with resultant
tendonitis and even rupture at the level of the posterior ankle joint
can be seen in dancers. Physical exam findings include:
posteromedial pain and crepitus, triggering of the great toe,
decreased passive extension of the great toe, and pain with
resisted great toe plantar flexion. Treatment is typically non-
operative, although Sammarco et al have demonstrated good
results with operative management of FHL tendonitis and tendon
rupture.
Dr.Wael Abboud (Foot and Ankle OB)

59-For the treatment of new onset plantar fasciitis, which of


the following modalities results in the highest patient
satisfaction at 8 weeks of follow-up?

QID: 3299

1. Achilles tendon–stretching program


2. Corticosteroid injection
3. Extracorporeal shock-wave therapy
4. Plantar fascia–specific stretching program
5. Distal tarsal tunnel decompression and partial plantar
fascia release

PREFERRED RESPONSE ▼ 4
CORRECT
Dr.Wael Abboud (Foot and Ankle OB)

A plantar fascia-specific stretching program has the highest patient


satisfaction at the 8 week follow-up interval. Symptoms of plantar
fasciitis include “start-up” inferior heel pain with patients often
preferring to walk on their toes for the first few steps when getting
out of bed. The pain lessens with ambulation and then increases
again with increased activity. Illustration A depicts a patient
performing plantar fascia-specific stretching. Illustration B displays
a patient performing an achilles tendon–stretching program.

The review article by Neufeld and Cerrato details that stretching


programs have been the primary treatment therapy modality for
patients with plantar fasciitis. The purpose of plantar fascia–
specific stretching is to recreate the windlass mechanism and
achieve tissue tension through a controlled stretch of the plantar
fascia.

The Level 2 study by Giovanni et al compared these 2 protocols


and showed that heel pain was eliminated or improved at 8 weeks
in 52% of patients treated with the plantar fascia–specific
stretching program versus only 22% of patients participating in the
Achilles tendon–stretching program. At 2-year follow-up, the study
reported no difference between the two groups with 92% of all
patients reporting total satisfaction or satisfaction with minor
reservations.
Corticosteroids(Answer 2) should be used rarely as they can
cause fat atrophy and even plantar fascia rupture. The FDA has
approved extracorporeal shock-wave therapy(Answer 3) for plantar
fasciitis lasting greater than 6 months. AAOS Comprehensive
Orthopaedic Review states surgical treatment(Answer 5) is
indicated for symptoms lasting greater than 9 months despite
conservative management.
Dr.Wael Abboud (Foot and Ankle OB)
Dr.Wael Abboud (Foot and Ankle OB)

60-A 34-year-old female has an insidious onset of heel pain


when first getting out of bed and at the end of the day after
prolonged standing. She works as a waitress and recently had
bariatric surgery with a current BMI of 35. She has a
gastrocnemius contracture noted on Silverskiold testing. AP
and oblique radiographs are shown in Figure A and lateral
radiograph is shown in Figure B. What is the most likely
diagnosis?

FIGURES: A B
Dr.Wael Abboud (Foot and Ankle OB)

1. Navicular stress fracture


2. Freiberg's Infraction
3. Plantar fasciitis
Dr.Wael Abboud (Foot and Ankle OB)

4. First branch of the lateral plantar nerve (Baxter's)


entrapment
5. Anterior tarsal tunnel syndrome

PREFERRED RESPONSE ▼ 3
CORRECT

Classic symptoms of plantar fasciitis include “start-up” inferior heel


pain with patients often preferring to walk on their toes for the first
few steps when getting out of bed. The pain lessens with
ambulation and then increases again with increased activity. None
of the other options have this classic constellation of symptoms.
Figures A and B do not show any evidence of fracture or
osteonecrosis.

The Level 3 case-control study by Riddle et al found that reduced


ankle dorsiflexion was the strongest independent risk factor for
development of plantar fasciitis with an odds-ratio(OR) of 23. BMI
>30 (OR=5)and work-related weight-bearing(OR=3) are
independent risk factors for plantar fasciitis.

61-A 44-year-old recreational runner began training for a half


marathon 6 weeks ago. Over the last week he has developed
heel pain that is worse in the morning upon awakening and
when he arises from his desk at the end of the workday.
Physical exam is notable for tenderness with direct palpation
of the anteromedial heel. Which of the following is the best
initial management?

QID: 148

1. Stretching of the achilles tendon and plantar fascia


along with a prefabricated shoe insert
2. Immobilization in a short leg cast
3. Steroid injection of the plantar fascia
4. Custom made orthotic with arch support
Dr.Wael Abboud (Foot and Ankle OB)

5. Surgical release of the medial third of the plantar fascia


origin

PREFERRED RESPONSE ▼ 1
CORRECT

The clinical presentation is consistent with plantar fasciitis. Initial


treatment includes stretching of the plantar fascia and achilles
tendon. Shoe inserts and heel cups may be beneficial in relieving
symptoms as well. Symptoms often take up to 6 months or a year
to resolve and surgical release of the plantar fascia should be
reserved for the exceptionally recalcitrant cases. Pfeffer et al
conducted a level 1 study of 236 patients with plantar fasciitis. All
patients were treated with achilles tendon and plantar fascia
stretching. The patients were then randomized to a custom orthotic
or prefabricated insert. The patients who received a prefabricated
insert demonstrated significantly greater improvement in clinical
symptoms at 8 weeks. Harty et al conducted a biomechanical
study of foot loading on live subjects. Lack of knee extension was
associated with prolonged loading of the forefoot. The authors
concluded that through the windlass mechanism of the plantar
fascia, hamstring tightness may predispose the foot to
development of plantar fasciitis. Illustration A demonstrates the
proper technique of plantar fascia stretching through stabilization
of the hindfoot and hyperextension of the toes. Illustration B
demonstrates an achilles tendon stretch that also incorporates
eccentric loading of the gastroc-soleus.
Dr.Wael Abboud (Foot and Ankle OB)

62-A 40-year-old female presents to the physician for an initial


visit with a 5-month history of plantar medial heel pain. She
notices it immediately on getting out of bed in the morning,
but the pain improves after a few steps. The pain is
exacerbated throughout her workday to the point where she is
unable to finish her work shift. Figure A shows a lateral
Dr.Wael Abboud (Foot and Ankle OB)

radiograph of the affected heel. Which of the following is the


most appropriate initial management?

FIGURES: A

1. Walker boot immobilization with full weightbearing for 4


weeks
2. Corticosteroid injection to the plantar fascia
3. Surgical release of 50% of the plantar fascia
4. Heel spur resection
5. Achilles stretching exercises

PREFERRED RESPONSE ▼ 5
CORRECT

This is the classic history of plantar fasciitis. The typical patient is


40-70 years of age and can point almost directly to the plantar
medial heel as the source from where the pain emanates. It is a
degenerative process associated with micro-tears of the plantar
fascia. Figure A shows a calcaneal spur. It is widely accepted that
heel spurs can occur concomitantly with plantar fasciitis, but they
Dr.Wael Abboud (Foot and Ankle OB)

are not the etiology of the disorder. Treatment is predominantly


aimed at intrinsic toe stretching (Illustration B) combined with
achilles stretching (Illustration A) as gastrocnemius contractures
are often found concomitantly. Night splints have also been found
to help. If surgery is required because conservative management
fails after 6-12 months, only the medial third of the plantar fascia is
released so that the medial longitudinal arch is not compromised.
Dr.Wael Abboud (Foot and Ankle OB)
Dr.Wael Abboud (Foot and Ankle OB)

63- The anterior drawer test most effectively tests for injury or
laxity or which of the following ligaments shown in Figure A?

FIGURES: A

1. A
2. B
3. C
4. D
5. E

PREFERRED RESPONSE ▼ 2
CORRECT

Answer choice B points to the anterior talofibular ligament (ATFL).


The ATFL is the most common ligament injured with ankle sprains
and occurs most often with the foot in slight plantar flexion. Plantar
flexion of the foot causes the ATFL to become parallel with the
axis of the foot and become taut. The orientation of the ATFL,
along with the other foot and ankle ligaments is seen in illustration
A. The ankle anterior drawer test can show injury of the ATFL and
Dr.Wael Abboud (Foot and Ankle OB)

is performed by doing an anterior drawer of the talus with the foot


in slight plantar flexion, and is seen in Illustration B. Illustration C,
shows a radiographic example of anterior translation of the talus
during an anterior drawer exam. Illustration D is a link to a video
demonstrating this exam maneuver.

In his study on the on the management of acute and chronic ankle


instability, Maffulli et al concluded that acute lateral ankle ligament
injuries are very common and can result in chronic instability if left
untreated. Conservative measures includes functional
rehabilitation which is the management of choice for acute and
chronic injuries. Surgical intervention is best reserved for high-
demand athletes.
Dr.Wael Abboud (Foot and Ankle OB)
Dr.Wael Abboud (Foot and Ankle OB)

64- A 21-year-old collegiate basketball player comes down


with a rebound and rolls his ankle. He is able to finish the
game, but complains of ankle pain and swelling afterwards.
Physical exam is notable for moderate inversion laxity with
the ankle held in dorsiflexion. With placement of the ankle in
plantarflexion, no inversion laxity is appreciated. Which of the
following ligaments has been attenuated?

QID: 3185

1. Anterior talofibular ligament


2. Calcaneofibular ligament
3. Anterior tibiofibular ligament
4. Posterior tibiofibular ligament
5. Deltoid ligament

PREFERRED RESPONSE ▼ 2
CORRECT
Dr.Wael Abboud (Foot and Ankle OB)

The primary static stabilizers of the lateral ankle are the anterior
talofibular ligament, calcaneofibular ligament, and posterior
talofibular ligament. The calcaneofibular ligament becomes most
taut with the ankle dorsiflexed and inverted. Conversely, the
anterior talofibular ligament is most tensioned with the ankle
plantarflexed and inverted. The anterior and posterior tibiofibular
ligaments contribute stability to the tibiofibular articulation and
syndesmosis. The deltoid ligament is the primary stabilizer
medially and is stressed with ankle eversion testing. Illustration A
is a depiction of the lateral ligaments of the ankle. Mafulli et al
reviews the evaluation and treatment of chronic ankle instability.

65- In the majority of patients, which of the following leads to


a good or excellent one-year prognosis in the treatment of
grade-III sprains of the lateral ligaments of the ankle?

QID: 779
Dr.Wael Abboud (Foot and Ankle OB)

1. Surgical repair of the ligament and casting


2. Casting alone
3. Early controlled mobilization
4. Application of a removable brace
5. All of the above result in good or excellent outcomes
one year after injury

PREFERRED RESPONSE ▼ 5
CORRECT

Kannus et al, in their CCR, summarized the results of 12


randomized studies which attempted to identify the proper
treatment of acute grade-III sprains of the lateral ligaments of the
ankle. They concluded that in 75-100% of patients, the one-year
prognosis was good or excellent irrespective of therapy (repair and
cast, cast or brace alone, or early controlled mobilization). In terms
of recovery time, however, they concluded that early controlled
mobilization provided the quickest recovery to full ROM and return
to work and physical activity.

66- A 38-year-old postal carrier complains of recurrent right


ankle sprains and lateral ankle pain. A clinical photograph
and radiograph are provided in Figures A and B. Coleman
block testing demonstrates correction of the deformity.
Custom orthotics, bracing, and NSAIDS have failed to provide
pain relief or prevent recurrent sprains. Which of the following
treatments should be pursued?

FIGURES: A B
Dr.Wael Abboud (Foot and Ankle OB)
Dr.Wael Abboud (Foot and Ankle OB)

1. Steroid injection of the sinus tarsi and taping of the


ankles before activity
2. Lateral ligament repair and augmentation with inferior
extensor retinaculum
3. Lateral ligament reconstruction with peroneus brevis
tendon grafting
4. First metatarsal osteotomy and lateral ligament
reconstruction with peroneus brevis tendon grafting
5. Triple arthrodesis and split peroneus brevis tendon graft
reconstruction of the lateral ligaments

PREFERRED RESPONSE ▼ 4
CORRECT

The photograph demonstrates bilateral cavovarus foot orientation.


Cavovarus positioning of the foot leaves the ankle susceptible to
inversion sprains and lateral ligament attenuation. The radiograph
shows a calcaneal pitch consistent with cavovarus deformity and
no evidence of degenerative change. Initial conservative treatment
should include orthotics with a recessed first metatarsal, lateral
forefoot wedge, reduced arch, and heel wedge. Failure of
conservative treatment is an indication for surgery. The lateral
ligament reconstruction must be augmented with an osteotomy to
correct the cavovarus deformity or it will be very prone to failure.
The flexible deformity (as demonstrated by Coleman block testing)
suggests successful correction with a metatarsal osteotomy as
opposed to a calcaneal osteotomy which would be reserved for a
rigid deformity.

Fortin et al review 13 cases of chronic lateral ankle instability with


cavovarus foot deformity. Ankles with significant degenerative
changes were treated with arthrodesis. Ankles without
degenerative changes were treated with lateral ligament
reconstruction with calcaneal and/or first metatarsal osteotomies to
correct the cavovarus deformity.

67- A 20-year-old female collegiate basketball player has had


recurrent ankle sprains of her right ankle. Trials of
immobilization and physical therapy have not prevented
further injuries. Physical exam reveals significant laxity of the
Dr.Wael Abboud (Foot and Ankle OB)

right ankle compared to the left ankle, but otherwise is


normal. Radiographs are unremarkable. What is the best
surgical treatment for this patient?

QID: 1083

1. Evans tenodesis (peroneus brevis tenodesis)


2. Modified Broström procedure
3. Allograft reconstruction with a tendon graft from the
fibula to the 5th metatarsal base
4. Primary ligament repair with lateralizing calcaneal
osteotomy
5. Primary ligament repair with a dorsiflexion osteotomy of
the 1st metatarsal

PREFERRED RESPONSE ▼ 2
CORRECT

When chronic lateral ligamentous instability remains despite


conservative treatment, the surgeon must choose between a
multitude of operative methods for surgical stabilization. For most
cases of chronic ankle instability in the athletic population, the
modified Brostrom technique (answer #1) is preferred. The
modified Brostrom procedure is an anatomic reconstruction of the
lateral ankle ligaments (ATFL and/or CFL) and augmentation with
the inferior extensor retinaculum. Messmer at al showed at a mean
follow-up of 34.5 months (minimum of 18 months), twenty patients
(91%) reported a good or excellent functional outcome with the
modified Brostrom procedure. Krips et al looked at anatomical
reconstruction and tenodesis in the treatment of chronic
anterolateral ankle instability in a retrospective multicenter study.
They found superior results in anatomical reconstruction compared
to tenodesis in regards to stability (anterior drawer test and talar tilt
& anterior talar subluxation on stress XR) and degenerative
changes. They concluded that when compared with anatomical
reconstruction, a tenodesis leads to inferior results in terms of
functional and mechanical stability, as well as overall satisfaction
at long-term follow-up. The Evans procedure (answer #1) is a
nonanatomic reconstruction as the tendon weave does not
Dr.Wael Abboud (Foot and Ankle OB)

recreate the ATFL or CFL but lies somewhere in between. It


involves harvesting either half or the entire peroneus brevis tendon
proximally and leaving it attached to the fifth metatarsal base
distally. The free proximal portion is then passed anterior to
posterior through a drill hole in the distal fibula or placed over the
anterior fibula and sutured to the periosteum.

Osteotomies are generally reserved when dealing with abnormal


foot shapes. Hindfoot varus can be treated with a valgus producing
calcaneal osteotomy. Hindfoot varus due to a plantar flexed 1st
metatarsal (confirmed with the Coleman block test) can be treated
with a dorsiflexion osteotomy of the 1st metatarsal.

68- In dancers, peroneal muscle weakness has been shown to


be the cause of which of the following?

QID: 89

1. Ankle sprain
2. Fibular fracture
3. Acute cuboid subluxation
4. Achilles rupture
5. Midfoot sprain

PREFERRED RESPONSE ▼ 1
CORRECT

The most common acute skeletal injury in the dancer is the


inversion sprain of the ankle and thought to be due to relative
peroneal muscle weakness. Sprains may occur in any ligament in
the foot or ankle, but most common ones involve the lateral
ligament complex (anterior talofibular, calcaneal-fibular, and
posterior talofibular ligaments); the anterior tibiofibular, the lateral
talocalcaneal ligament; and, occasionally the medial (deltoid)
ligament. Regardless of the method of treatment, adequate
physical therapy and proper rehabilitation are necessary to restore
normal use following injury. Restoration of full peroneal strength is
essential to minimize risk of recurrent sprains.
Dr.Wael Abboud (Foot and Ankle OB)

The reference by Hamilton is a review of foot and ankle injuries in


the dancing population. He reviews the importance of early
diagnosis and covers the different conservative and operative
treatments available.

69- A football player develops a synostosis between the distal


tibia and fibula 8 months following treatment for a high ankle
sprain. What is the most appropriate indication and timing for
surgical excision?

QID: 1387

1. Chronic lateral ankle instability at any time


2. Progressive loss of plantar-flexion at any time
3. Progressive loss of plantar-flexion and increased uptake
on bone scan
4. Persistent pain despite non-operative treatment and no
uptake on bone scan
5. Persistent pain despite non-operative treatment and
"hot" on bone scan

PREFERRED RESPONSE ▼ 4
CORRECT

Post-traumatic tibiofibular synostosis may occur following a high


ankle sprain where the interosseous membrane was disrupted.
The heterotopic ossification usually develops within 6 to 12
months. Typically, patients are able to return to sports despite the
lack of normal ankle dorsiflexion and mobility between the tibia and
fibula. Surgical excision is reserved for persistent pain that fails to
respond to nonsurgical management once the ossification is “cold”
(does not show increased uptake) on bone scan.

Whiteside et al described the first case series back in 1978 with 6


professional athletes who developed a tib-fib synostosis following
high ankle sprain. Henry subsequently described it in professional
basketball players.

Albers et al reported on 15 synostoses following 230 operatively


Dr.Wael Abboud (Foot and Ankle OB)

treated ankle fracture. These were usually Weber C fractures and


their result at final follow-up was similar to other patients without
synostoses.

70- A 21-year-old male collegiate basketball player presents


with 1 year of left lower leg pain. The pain is worse with
activity and the leg is tender to palpation. He denies
constitutional symptoms, and conservative treatment has
failed to provide relief. He denies recent trauma, but did
sustain a severe ankle sprain 7 years ago. Radiograph, bone
scan, and CT scans are shown in Figures A-D. What is the
next most appropriate step in management?

FIGURES: A B C D
Dr.Wael Abboud (Foot and Ankle OB)
Dr.Wael Abboud (Foot and Ankle OB)
Dr.Wael Abboud (Foot and Ankle OB)
Dr.Wael Abboud (Foot and Ankle OB)

1. External beam irradiation with 60Gray to lesion


2. Neoadjuvant multiagent chemotherapy followed by
surgical resection of lesion followed by adjuvant multiagent
chemotherapy
3. Observation and repeat bone scans to plan external
beam irradiation of 700cGray to lesion
4. Indomethacin 25 mg PO tid for 6 weeks
5. Observation and repeat bone scans to plan surgical
resection

PREFERRED RESPONSE ▼ 5
CORRECT
Dr.Wael Abboud (Foot and Ankle OB)

This patients history and imaging studies are consistent with


tibiofibular synostosis.

Tibiofibular synostosis is a rare cause of leg pain in athletes that


usually presents with anterior shin pain for one or two years as
shown in the case series by Henry et al of 2 professional
basketball players. The review of these 2 patients found the
synostosis was a result of a prior tibia stress fracture and both
patients were treated symptomatically without surgery.

The review article by Pell et al notes that synostoses are often


asymptomatic however, when nonsurgical measures fail in
symptomatic patients, excision of the synostosis through an
anterolateral approach may be required. Bone scan is indicated if
considering surgical excision, and it should demonstrate
ossification is complete (no uptake) before surgical excision is
performed. Irradiation of 700cGray and Indomethacin 25 mg oral
TID for 6 weeks are treatments for heterotopic ossification
prophylaxis following acetabular fracture surgery, but there is no
literature documenting the use of this treatment for chronic
synostoses. Chemotherapy regimens should only be initiated when
neoplasm is identified on biopsy. External beam irradiation can be
utilized in soft tissue sarcomas, lymphoma, multiple myeloma,
however at a total dose of 60Gray the patient is unlikely to heal
any surgical wounds.

71- Which test for syndesmotic injury of the ankle has the
fewest false-positive results and smallest inter-observer
variance?

QID: 974

1. Squeeze test
2. Fibular translation
3. Cotton test
4. External rotation stress test
5. Anterior drawer

PREFERRED RESPONSE ▼ 4
Dr.Wael Abboud (Foot and Ankle OB)

CORRECT

The external rotation stress test helps in diagnosing syndesmotic


injuries. The athlete's knee is flexed 90 degrees and the ankle is in
neutral. Stabilizing the tibia and fibula with one hand, the examiner
externally rotates the ankle with the other. Pain over the
syndesmosis indicates a positive test. Beumer et al tested the
squeeze, fibula translation, Cotton, and external rotation tests.
None of the syndesmotic tests was uniformly positive in chronic
syndesmotic injury. The external rotation test had the fewest false-
positive results, the fibula translation test the most. The external
rotation test had the smallest inter-observer variance.

72- A 38-year-old competitive slalom skier is making a turn to


the left around a pole. The right ski sticks in the snow as
shown in Figure A, causing external rotation of the right ski
and boot. Which of the following ankle ligaments is most
likely to be the initial structure injured?

FIGURES: A
Dr.Wael Abboud (Foot and Ankle OB)

1. Calcaneofibular ligament
2. Anterior inferior tibiofibular ligament
3. Deep deltoid ligament
4. Superficial deltoid ligament
5. Anterior talofibular ligament

PREFERRED RESPONSE ▼ 2
CORRECT

High ankle sprains are external rotation injuries of the ankle and
syndesmosis. They often occur in competitive slalom skiers, and
the anterior inferior tibifibular ligament is the initial ligament injured.
External rotation of the foot on the leg causes the talus to press
against the lateral malleolus. This rotational movement first affects
the anterior inferior tibiofibular ligament of the syndesmosis. If
external rotation continues, the interosseous membrane and then
the posterior tibiofibular ligament will be injured.

The review article by Clanton indicates the anterior inferior


tibiofibular ligament is the most commonly injured ligament in ankle
sprains where the mechanism is of injury is external rotation. This
occurs regardless of the position of the foot at the time of injury.
Pure dorsiflexion causes the interosseus ligaments to tighten and
abduction on a neutral ankle can cause interosseus injury when
preceded by deltoid injury or medial malleolus fracture

73- A 20-year-old male sustains a right foot injury after a


head-on motor vehicle collision. He is unable to place weight
on the foot to ambulate. His pain is exacerbated with
abduction of the midfoot. He is neurovascularly intact in the
foot. Non-weightbearing radiographs are demonstrated in
Figures A-B and a clinical image of the foot is shown in Figure
C. What is the most appropriate next step in management?

FIGURES: A B C
Dr.Wael Abboud (Foot and Ankle OB)
Dr.Wael Abboud (Foot and Ankle OB)

1. Open reduction and arthrodesis of the medial two


tarsometatarsal joints
2. External fixation of the foot followed with staged open
reduction and screw fixation across the medial two
tarsometatarsal joints
3. Open reduction and screw fixation across the medial
two tarsometatarsal joints
4. Debridement of Morel-Lavallee lesion and external
fixation of the foot followed with staged open reduction and
screw fixation across the medial two tarsometatarsal joints
5. Open reduction and screw fixation across the medial
two tarsometatarsal joints with anatomic ligamentous
reconstruction

PREFERRED RESPONSE ▼ 1
Dr.Wael Abboud (Foot and Ankle OB)

CORRECT

The history, clinical images, and radiographs are consistent with a


Lisfranc injury. Instability of the Lisfranc joint is the result of injury
to both the interosseous first cuneiform-second metatarsal
ligament (Lisfranc's ligament) and the plantar ligament between
the first cuneiform and the second and third metatarsals.

Ly and Coetzee performed a Level 1 investigation of 41 patients


with an isolated acute or subacute Lisfranc dislocation treated with
ORIF or primary arthrodesis of the medial column of midfoot. Two
years postoperatively, the mean AOFAS Midfoot score was 68.6
points in the open-reduction group and 88 points in the arthrodesis
group (p < 0.005). Postoperative level of activities was significantly
higher in the arthrodesis group and 25% of the ORIF group
required a subsequent revision to an arthrodesis. They concluded
that primary arthrodesis of the medial two or three rays has a
better short and medium-term outcome than ORIF.

Henning et al conducted a Level 1 investigation of 40 patients with


a Lisfranc fracture/dislocation treated with either ORIF or primary
arthrodesis. They found that nearly 80% of the ORIF group needed
subsequent hardware removal compared to 15% of the arthrodesis
group. They found no statistical difference in functional outcomes
at nearly 4 year follow-up. They concluded that there is no
significant difference in SF-36 and Short Musculoskeletal Function
Assessment outcome scores when compared to ORIF.

The deep neurovascular bundle (as shown in Illustration A) should


be protected following its identification after skin incision .
Illustration B is an example of a tarsometatarsal arthrodesis.
Dr.Wael Abboud (Foot and Ankle OB)
Dr.Wael Abboud (Foot and Ankle OB)

74- A 24-year-old man sustains an injury to the left foot.


Stress radiographs are seen in Figure A. Injury to which
ligament or ligaments are needed to produce the transverse
instability seen here?

FIGURES: A

1. Spring ligament and bifurcate ligament


2. Interosseous first cuneiform-second metatarsal ligament
3. Interosseous first cuneiform-second metatarsal ligament
and plantar ligament between the first cuneiform and the
second and third metatarsals
4. Bifurcate ligament and Interosseous first cuneiform-
second metatarsal ligament
Dr.Wael Abboud (Foot and Ankle OB)

5. Long plantar ligament and plantar ligament between the


first cuneiform and the second and third metatarsals

PREFERRED RESPONSE ▼ 3
CORRECT

Transverse instability of the Lisfranc joint is the result of injury to


both the interosseous first cuneiform-second metatarsal ligament
(Lisfranc's ligament) and the plantar ligament between the first
cuneiform and the second and third metatarsals. Kaar et al
performed a cadaveric study in which sequential sectioning of the
Lisfranc ligament followed by the plantar ligament between the first
cuneiform and the second and third metatarsals was performed in
order to simulate a low energy Lisfranc injury with transverse
instability . Stress abduction radiographs were positive for all
specimens in which both ligaments were sectioned, while
sectioning of just the Lisfranc ligament was positive for only one
specimen. The other options would not lead to a Lisfranc injury
with transverse instability.

75- As an alternative to open reduction and internal fixation


for the injury pattern seen in Figure A, what alternative
treatment has been shown to be more effective?

FIGURES: A B
Dr.Wael Abboud (Foot and Ankle OB)

1. open reduction and primary arthrodesis of the medial


and middle column
2. open reduction and primary arthrodesis of the medial,
middle, and lateral column
Dr.Wael Abboud (Foot and Ankle OB)

3. closed percutanous pinning


4. closed reduction and casting
5. partial weight bearing in removable boot

PREFERRED RESPONSE ▼ 1
CORRECT

The lateral radiographs shows a ligamentous Lisfranc injury. There


does not need to be a fracture to be called a Lisfranc injury; and
the ligamentous ones typically have even worse outcomes than
similar fracture-type injuries. These uniformly have poor, painful
outcomes if treated non-operatively because recurrence of the
dislocation occurs without rigid bony fixation to allow soft tissues to
heal. Typically, these have been treated with ORIF, but outcomes
are often poor and many times the deformity reccurs because the
soft tissues may not heal strong enough to resist weight bearing
forces, as compared to fractures which are strong once healed.

Ly et al. published a prospective, randomized study of ORIF alone


versus ORIF with primary arthrodesis for patients with primarily
ligamentous Lisfranc injuries. In the ORIF alone group, 16 of 20
had revision surgery, with only 4 of 21 needed revision surgery in
the fusion group, and the fusion group had significantly better
functional outcomes at 2 yrs.

The referenced study by Mulier et al compared ORIF versus


arthrodesis and noted the ORIF group had less pain at final follow-
up, with no differences in subsequent revision surgery. Stiffness of
the forefoot, loss of metatarsal arch, and sympathetic dystrophy
occurred more frequently in the complete arthrodesis group. They
concluded that primary complete arthrodesis should be reserved
as a salvage procedure.

76- Which ligament connects the medial cuneiform to the


base of the second metatarsal?

QID: 608
Dr.Wael Abboud (Foot and Ankle OB)

1. Spring ligament
2. Chopart ligament
3. Lisfranc ligament
4. Intermetatarsal ligament
5. Calcaneofibular ligament

PREFERRED RESPONSE ▼ 3
CORRECT

The Lisfranc ligament arises from the lateral surface of the medial
cuneiform and inserts onto the medial aspect of the second
metatarsal base near the plantar surface. It is the largest and
strongest interosseous ligament in the tarsometatarsal joint
complex. The spring ligament (plantar calcaneonavicular ligament)
is a broad, thick band of fibers, which connects the anterior margin
of the calcaneus to the navicular. It supports the head of the talus
and helps maintain the medial longitudinal arch of the foot.
Dr.Wael Abboud (Foot and Ankle OB)

77- A 37-year-old man was involved in a high velocity motor


vehicle accident 6 months ago. He spent 4 months in the ICU
recovering from a severe head injury. He has now transitioned
to a rehabilitation hospital and complains of left foot pain that
becomes severe with weight bearing and attempted
ambulation. Radiographs are provided in figures A-C. Which
of the following is the best management?

FIGURES: A B C
Dr.Wael Abboud (Foot and Ankle OB)
Dr.Wael Abboud (Foot and Ankle OB)
Dr.Wael Abboud (Foot and Ankle OB)

1. Custom orthotics and physical therapy


2. Tarsometatarsal closed reduction and percutaneous
screw fixation
3. Tarsometatarsal open reduction internal fixation
4. Tarsometarsal arthrodesis
5. Tarsometatarsal arthrodesis and triple arthrodesis

PREFERRED RESPONSE ▼ 4
CORRECT

The clinical presentation and radiographs present a Lisfranc


(tarsometatarsal dislocation) injury that is 6 months old. The first,
second, and third tarsometatarsal articulations are involved. Given
the chronicity of the injury, arthrodesis is the best treatment option
available to relieve pain and improve function. There is no
indication to include a triple arthrodesis with the tarsometatarsal
arthrodesis.

The Mulier article compared total arthrodesis (medial + lateral) vs


medial column arthrodesis with lateral column ORIF for severe
Lisfranc injuries. They found inferior outcomes with total
arthrodesis.

Komenda et al reviewed 32 patients who underwent


tarsometatarsal arthrodesis at a minimum of 6 months following
injury. The patients improved significantly compared to the
preoperative pain and function. However, a number of
complications were encountered including neuritis, malunion,
nonunion, wound sloughing, and RSD.

78- A 30-year-old equestrian caught her foot in a stirrup 1


week ago and now complains of midfoot pain with difficulty
bearing weight. Radiographs are shown in figure A. What
treatment is most appropriate?

FIGURES: A
Dr.Wael Abboud (Foot and Ankle OB)

1. non-weight bearing in an aircast


2. weight bearing as tolerated in a walking cast
3. modified-Brostrom procedure
4. extensor hallucis longus repair
5. open reduction internal fixation

PREFERRED RESPONSE ▼ 5
CORRECT

The patient has a Lisfranc injury. These are typically high energy
injuries involving the Lisfranc ligament which connects the base of
the 2nd metatarsal to the medial cuneiform. Dorsal dislocation is
most common form. Anatomic reduction is necessary and can only
reliably be achieved through open reduction and internal fixation.
The ligament or a bony avulsion can become incarcerated in the
joint preventing anatomic reduction. Following surgery patients
should be treated with protected weight-bearing for 3-5 months
Dr.Wael Abboud (Foot and Ankle OB)

and therapy emphasizing passive midfoot ROM. Midfoot post


traumatic arthritis and pain are the long term outcomes of a non-
reduced joint. Kuo et al showed less arthritis and better AOFAS
scores with anatomic ORIF.

79- A 54-year-old male sustained a right foot injury two years


ago in a motor vehicle collision. The patient reports he was
treated for a ligament injury in his foot with a non-
weightbearing short leg cast for 2 months. Physical
examination reveals no signs of infection and full sensation
and motor strength in the foot. During gait examination he
has pain during push-off of the right foot and loss of medial
longitudinal arch height in the stance phase. A radiograph
obtained at the time of initial injury is shown in Figure A. What
is the most appropriate next step in management?

FIGURES: A
Dr.Wael Abboud (Foot and Ankle OB)

1. Lateral column lengthening procedure (Evans)


2. Tarsometatarsal joint arthrodesis
3. Talonavicular joint arthrodesis
4. Lateral wedge closing calcaneal osteotomy (Dwyer)
5. Subtalar, talonavicular, and calcaneocuboid joint
arthrodesis (Triple)

PREFERRED RESPONSE ▼ 2
CORRECT

The patient's clinical presentation is consistent with a chronic


Lisfranc injury with posttraumatic midfoot arthritis. Figure A shows
an acute Lisfranc injury with diastatis between the medial
cuneiform and 2nd metatarsal.

The Lisfranc joint complex consists of tarsometatarsal,


intermetatarsal, and intertarsal articulations. The Lisfranc ligament
goes from medial cuneiform to base of 2nd metatarsal on plantar
surface and provides transverse foot stability.

The Level 5 review article by Thompson and Mormino state that


shoe inserts/modifications and nonsteroidal anti-inflammatory
medications are the mainstay of non-surgical treatment for
posttraumatic arthritis after Lisfranc injury. If these modalities fail,
arthrodesis of the affected joints is the treatment of choice.

Illustration A shows the anatomic postion of the stout, plantar


portion Lisfranc ligament with the deep band spanning from the
medial cuneiform to the 2nd metatarsal and the superficial band
extending to the 3rd metatarsal
Dr.Wael Abboud (Foot and Ankle OB)

80- A 54-year-old woman sustains a twisting injury to her foot.


A radiograph is provided in Figure A. What is the next step in
management?

FIGURES: A
Dr.Wael Abboud (Foot and Ankle OB)

1. closed reduction and pinning


2. closed reduction and casting
3. open reduction and internal fixation
4. CAM walker and weight bearing as tolerated
5. weight bearing as tolerated

PREFERRED RESPONSE ▼ 3
CORRECT

This is a Lisfranc injury. No consensus exists about optimal


treatment of these injuries, but evidence is clear that a satisfactory
result is directly related to the accuracy of the reduction and its
successful maintenance through healing. Some recent studies
support primary fusion over ORIF as optimal treatment for purely
ligamentous injuries; however, this is not an answer choice. Many
Dr.Wael Abboud (Foot and Ankle OB)

authors recommend open reduction and screw fixation for


treatment of all TMT fractures and dislocations. Alterations in the
anatomy of the Lisfranc joints secondary to trauma can result in
foot collapse and altered weight bearing which are difficult to
salvage later.

81-A 21-year old college lacrosse player injures left her foot
while walking down a flight of stairs. She has pain and
inability to bear weight on her injured foot. She has no plantar
ecchymosis but does have tenderness over her lateral foot. A
radiograph of her foot is found in Figure A. What is the best
form of management?

FIGURES: A

1. Hard-soled shoe
2. Cast immobilization
Dr.Wael Abboud (Foot and Ankle OB)

3. Modified Brostrom procedure


4. Intramedullary screw fixation
5. Operative repair of the lisfranc fracture

PREFERRED RESPONSE ▼ 4

The clinical scenario and radiographs are consistent with a fracture


of the base of the fifth metatarsal. In young athletes, operative
screw fixation is the treatment of choice. The 5th metatarsal is
divided into 3 zones. Zone I is an avulsion fracture, zone II is
described as a jones fracture and zone III is proximal diaphyseal
fracture (Illustration A). Nonunions are more common with
fractures in zones II and III.

Lehman et al reviewed fractures of the base of the fifth metatarsal.


They describe the current controversies regarding nomenclature
and treatment. They recommend NWB immobilization for acute
fractures and delayed unions. The active patient and non-unions
are treated operatively.
Dr.Wael Abboud (Foot and Ankle OB)

82-A 20-year-old male collegiate basketball player presents


with a 1 day history of left foot pain. He developed severe pain
on the lateral border of his left foot after landing from a jump.
The pain is worsened with weightbearing and walking. The
patient reports that 12 weeks ago he sustained a similar injury
and underwent surgery on his foot by a different surgeon. He
reports that his physician released him to full activity 4 weeks
ago because he had no pain. He is currently tender to
palpation on the lateral border of the foot. Radiographs and
CT scan are shown in Figures A-D. What is the most
appropriate next step in management.

FIGURES: A B C D
Dr.Wael Abboud (Foot and Ankle OB)
Dr.Wael Abboud (Foot and Ankle OB)

1. Retain hardware and bone graft fracture site


2. Immobilization in a cast for 4 weeks with non-weight
bearing precautions
3. Immobilization with a CAM walker boot with weight
bearing to tolerance
4. Exchange intramedullary screw fixation
5. External bone stimulation with ultrasound

PREFERRED RESPONSE ▼ 4

Radiographic views demonstrate a bent 4.0 mm cannulated


intramedullary screw along with incomplete healing of the left
proximal 5th metatarsal fracture. The most appropriate treatment
for the bent, failed hardware is removal of the hardware and
placement of a larger intramedullary screw (Illustrations A and B).
Dr.Wael Abboud (Foot and Ankle OB)

It is important to note that return to full activity prior to radiographic


union is directly related to the risk of potential treatment failure.

The study by Larson et al examined 15 patients (mean age 21.7


years) who underwent cannulated screw fixation of a Jones
fracture between 1993 and 1999. There were 6 failures: four
refractures and two symptomatic nonunions. The mean time to full
activity was 6.8 weeks for the patients with failure, compared with
9 weeks for patients who did not have complications. Although all
patients were asymptomatic and radiographically progressing to
union before return to full activity, only one of 6 patients with
failures had complete radiographic union, compared with 6 of 7
patients with no complications. There was a higher proportion of
elite athletes among the failure group (83%) compared with those
without complications (11%). This study showed no significant
differences in age, sex, screw diameter, use of bone graft, or age
of fracture between patients with failures and those without
complications. However, it should be noted that other studies have
reported that smaller sized and cannulated screws may be risk
factors for treatment failure.
Dr.Wael Abboud (Foot and Ankle OB)

83-A 55 year-old woman comes to you with 2 months of right


foot pain. She is active in ballet and her pain is exacerbated
with push-off and en pointe maneuvers. Radiograph, bone
Dr.Wael Abboud (Foot and Ankle OB)

scan, and MRI are found in Figures A-C, respectively. Your


next step in management should consist of:

FIGURES: A B C
Dr.Wael Abboud (Foot and Ankle OB)
Dr.Wael Abboud (Foot and Ankle OB)

1. Percutaneous biopsy and referral to an orthopaedic


oncologist
2. Walker boot application and evaluation for metabolic
bone disease
3. Referral to an orthopaedic oncologist for limb salvage
procedure
4. Internal fixation of the fracture and evaluation for
metabolic bone disease
5. Metatarsal-cuneiform fusion of the Lisfranc joint

PREFERRED RESPONSE ▼ 2
CORRECT

Stress fracture of the second metatarsal is classically described in


amenorrheal dancers. However, this patient is of an age that a
fragility fracture should be considered. Metatarsal stress fractures
can be successfully treated with a short-leg cast, low walking boot,
or hard-soled shoe with arch support. MRI is a useful screening
Dr.Wael Abboud (Foot and Ankle OB)

tool, but CT better visualizes bone when the fracture is visible on


plain radiographs. The reference by Bogoch et al created a
multidisciplinary team to enroll a cohort of patients at risk for
osteoporosis. They found that >95% of patients were appropriately
diagnosed, treated, or referred for osteoporosis care with use of
this team.

84-A 19-year-old cross country runner complains of 3 months


of foot pain with running. Based on the radiographs shown in
Figure A, what is the most appropriate next step in treatment?

FIGURES: A

1. open reduction, internal fixation


2. excisional biopsy
3. continue running with a molded orthotic
Dr.Wael Abboud (Foot and Ankle OB)

4. protected weightbearing with crutches, with slow return


to running
5. percutaneous Kirschner wire fixation

PREFERRED RESPONSE ▼ 4
CORRECT

The radiographs show a diaphyseal stress fracture of the 3rd


metatarsal with obvious callus formation and good alignment of the
bone, coined a march fracture and occur in the 2nd, 3rd and 4th
metatarsals, with 2nd being the most common. The cited articles
indicate that a technium99 bone scan is the most useful diagnostic
imaging modality. Treatment consists of 4-6 weeks of rest with
gradual onset in activity to pre-injury activity level. The other
treatments are not appropriate for stress fractures.

85-A 26-year-old professional ballet dancer presents with


insidious onset of right midfoot pain which began 4 weeks
ago. She has no history of ankle or foot trauma, and medical
history is significant only for delayed menarche. Radiographs
are shown in Figure A. What is the most likely diagnosis?

FIGURES: A B
Dr.Wael Abboud (Foot and Ankle OB)
Dr.Wael Abboud (Foot and Ankle OB)

1. Lisfranc joint injury


2. Navicular stress fracture
3. Second metatarsal base stress fracture
4. Plantar fascia strain
5. First metatarsal base stress fracture

PREFERRED RESPONSE ▼ 3
CORRECT

Stress fractures in ballet dancers occur most frequently at the


second metatarsal base. The radiographs shown in Figures A and
B demonstrate periosteal stress reaction typical of these injuries.
The radiographs also show a relatively long second metatarsal
compared to the first, which can be a risk factor for the
development of a second metatarsal stress fracture. MRI, shown in
Illustration A and B, can be obtained to help confirm the diagnosis.
O'Malley et al identified 51 professional dancers (64 fractures) who
Dr.Wael Abboud (Foot and Ankle OB)

sustained a stress fracture at the base of the second metatarsal.


Delayed menarche was common in those affected, and the usual
location of the fracture was at the proximal metaphyseal-
diaphyseal junction of the second metatarsal. Treatment consisted
of a short leg walking cast for 6 patients, and a wooden shoe and
symptomatic treatment for the remainder. The patients returned to
performance at an average of 6.2 weeks following diagnosis, with
14% reporting continued pain with dancing. No patients required
bone grafting for persistent symptoms.
Dr.Wael Abboud (Foot and Ankle OB)

86-A 24-year-old female who is training for her first marathon


presents with six weeks of increasing foot pain. An AP
radiograph and representative axial cut of her CT scan of her
Dr.Wael Abboud (Foot and Ankle OB)

injury are seen in figures A and B. Management should


consist of which of the following?

FIGURES: A B

1. Weight bearing as tolerated in a hard soled shoe


2. Non weight bearing cast immobilization
3. Fragment excision and posterior tibial tendon
advancement
4. Percutaneous screw fixation
5. Open reduction with autologous bone graft

PREFERRED RESPONSE ▼ 2
CORRECT

The patient presents with symptoms and imaging studies


consistent with a navicular stress fracture. Initial mangement of
these injuries consists of non weight bearing cast immobilization.

Khan et al found that patients managed with a minimum of 6


weeks of non weight bearing (NWB) had significantly improved
rates of return to sport (86%) compared with patients that were
allowed to weight bear as tolerated (26%). After failure of weight
bearing management, 6/7 patients who were then NWB in a cast
were able to return to sports.

Lee et al review the clinical presentation, evaluation, and treatment


of navicular stress fractures. The authors advocate for initial
management with NWB immobilization and note that CT and MRI
Dr.Wael Abboud (Foot and Ankle OB)

may be useful to determine the chronicity of the injury.

Torg et al performed a meta-analysis of the existing literature


regarding treatment of navicular stress fractures. The authors note
that no difference has been seen in patient outcomes between
NWB and operative management and they state that NWB cast
immobilization should be the initial treatment for navicular stress
fractures.

Incorrect Answers:
Answer 1: Weight bearing as tolerated in a hard soled shoe does
is less effective than NWB cast immobilization for navicular stress
fractures
Answer 3: Fragment excision and tendon advancement is not a
described technique to manage these injuries
Answer 4: Percutaneous screw fixation may be indicated after
failure of 6-8 weeks of non weight bearing
Answer 5: Open reduction, with or without bone grafting, is not the
preferred initial management

87-A 30-year-old professional ballet dancer presents with


persistant ankle pain after an ankle sprain 6 months ago.
Physical therapy and NSAID's have not alleviated the
symptoms. Physical exam reveals some joint swelling but no
ligamentous instability. Radiographs are unremarkable. What
is the next appropriate step in the management of this
patient?

QID: 224

1. Continue physical therapy


2. Avoidance of dancing
3. MRI
4. Ankle steroid injection
5. Diagnostic ankle arthroscopy

PREFERRED RESPONSE ▼ 3
CORRECT
Dr.Wael Abboud (Foot and Ankle OB)

The vast majority of ankle sprains heal well with time, rest,
therapy, and temporary immobilization. In those approximate 10%
that do not improve, an osteochondral lesion of the talus and
persistent instability must be considered. The question stem states
that there is no ligamentous instability so the next step should be
an MRI to evaluate for an osteochondral lesion of the talus (OLT).
Surgery is indicated for OLTs if conservative therapy fails after 6
months. Tol et al performed a systematic review of 32 articles and
showed that excision, curettage, and drilling had the highest
success rate (85%), followed by excision and curettage (78%).
Nonop (45%) and excision only (38%) were less successful and
not recommended. The reference by Barnes and Ferkel is a review
of the evaluation and treatment of OLT's.

88-The current body of available evidence supports the use of


ankle arthroscopy for all of the following indications EXCEPT:

QID: 584

1. Ankle arthrodesis
2. Debridement of degenerative ankle cartilage
3. Osteochondral lesions
4. Anterior ankle impingement
5. Loose body removal

PREFERRED RESPONSE ▼ 2

Ankle arthroscopy has evolving indications, but the referenced


study by Glazebrook et al found fair evidence-based literature to
support a recommendation for the use of ankle arthroscopy for the
treatment of osteochondral lesions and for ankle arthrodesis.
Arthroscopy is also indicated in anterior ankle impingement as
shown in the preoperative and postoperative radiographs shown in
Illustration A and B. Ankle arthroscopy for ankle instability, septic
arthritis, arthrofibrosis, and removal of loose bodies was supported
with only limited evidence. Treatment of ankle arthritis, excluding
isolated bony impingement, was not effective and therefore this
indication was not recommended.
Dr.Wael Abboud (Foot and Ankle OB)
Dr.Wael Abboud (Foot and Ankle OB)

89-A 28-year-old rugby player has had anterior ankle pain for
several years. On physical exam he has painful and limited
dorsiflexion of the ankle. Based on a dorsiflexed ankle
radiograph shown in figure A, what is the most appropriate
treatment?

FIGURES: A

1. Arthroscopic chondral drilling


2. Arthroscopic tibial debridement
3. Modified Brostrom procedure
4. Arthroscopic os trigonum excision
5. Arthroscopic tibiotalar arthrodesis

PREFERRED RESPONSE ▼ 2
CORRECT
Dr.Wael Abboud (Foot and Ankle OB)

The exam and xrays reveal tibiotalar anterior impingement without


subtalar involvement. Tibiotalar impingement during dorsiflexion
with concomitant mild arthritis can be treated with anterior ankle
cheilectomy/debridement (arthroscopic or open). Ankle arthrodesis
with cancellous screws remains the gold standard for surgical
treatment of more severe ankle arthritis . Ankle arthroplasty is
another option however it is technically demanding with potential
complications. Illustration A shows a radiograph following
arthroscopic anterior tibiotalar debridement.

Level 4 evidence from Monroe et al found a 93% rate of tibiotalar


fusion using cancellous screws for fixation. Holt et al also found a
93% rate of fusion in low risk patients and 74% fusion rate overall.
Similarly, Dennis et al found a 94% rate of fusion using cancellous
screws for internal fixation.

90-The single rocker sole shoe modification is best indicated


for relief of pain in patients with what foot or ankle pathology?
Dr.Wael Abboud (Foot and Ankle OB)

QID: 3433

1. Midfoot prominences associated with Charcot


arthropathy
2. Severe toe-tip ulcerations
3. End-stage tibiotalar arthritis with limited motion
4. Fixed ankle dorsiflexion deformity
5. Fixed planovalgus deformity

PREFERRED RESPONSE ▼ 3
CORRECT

A single rocker sole shoe modification (Illustration A) is intended to


increase propulsion at toe-off, decrease pressure on heel strike,
and reduce the need for ankle motion. It is indicated for use in
patients with severe tibiotalar or subtalar arthritis, or those status-
post fusion of these joints.

Janisse et al review shoe modifications and foot orthoses as they


relate to the nonsurgical management of foot and ankle pathology.
With regards to rocker soles, they describe the six-types and how
each is specifically used for an individual patient's foot problem. In
general, the biomechanical effects of the rocker soles are restoring
lost motion in the foot and ankle and off-loading plantar pressure
on some part of the foot. Use of a rocker sole can result in overall
improvement in gait.

incorrect answers:
1-Midfoot prominences are best relieved with double rocker soles
(Illustration B), which offload this region of the foot with weight
bearing.
2-Although all rocker sole shoes relieve forefoot pressure to some
degree, severe toe-tip ulcerations in particular achieve the greatest
relief with the use of a severe angle rocker sole (Illustration C).
4-Fixed ankle dorsiflexion deformities are best accomodated with a
negative heel rocker sole (Illustration D).
5-Rocker sole shoe modifications are not typically used in the
treatment of fixed planovalgus deformities.
Dr.Wael Abboud (Foot and Ankle OB)
Dr.Wael Abboud (Foot and Ankle OB)

91-A 57-year-old male has right ankle pain for 6 years and has
failed conservative management. Radiographs of the ankle
are shown in Figures A and B. Which of the following is a
contraindication for a total ankle arthroplasty?

FIGURES: A B
Dr.Wael Abboud (Foot and Ankle OB)
Dr.Wael Abboud (Foot and Ankle OB)

1. Posttraumatic arthritis
2. Reconstructible ankle ligament damage
3. Neuropathic joint disease
4. Inflammatory arthritis
5. Age greater than 50 years old

PREFERRED RESPONSE ▼ 3
CORRECT

Total ankle arthroplasty (TAA) is an alternative to arthrodesis,


especially for elderly patients with end-stage arthritis and normal
ankle and hindfoot alignment.

According to the paper by Deorio et al, the indications for TAA


include primarily posttraumatic and inflammatory arthritis.
Contraindications to TAA include unresectable osteonecrotic bone,
peripheral vascular disease, neuropathy, neuropathic joint disease
Dr.Wael Abboud (Foot and Ankle OB)

(Charcot arthropathy), ankle infection, severe joint laxity,


nonreconstructible ankle ligaments, loss of lower leg muscular
control, and severe osteopenia or osteoporosis

92-At long-term follow-up, patients undergoing the procedure


shown in Figure A have been shown to have significant rates
of findings of which of the following?

FIGURES: A

1. Need for revision surgery


2. Hardware failure
3. Need for crutches or walker
4. Ipsilateral knee and/or hip degenerative changes
5. Ipsilateral midfoot and/or hindfoot degenerative changes

PREFERRED RESPONSE ▼ 5
Dr.Wael Abboud (Foot and Ankle OB)

CORRECT

Figure A shows a radiograph of an ankle arthrodesis. Long term


follow-up has shown the increased rate of ipsilateral midfoot and
hindfoot arthritis, hypothesized to be secondary to the increased
stresses seen at these joints with loss of ankle range of motion.
The referenced study by Thomas et al concluded that ankle fusion
provides good pain relief and improves overall function, but it is a
salvage procedure and can cause persistent alterations in gait with
a potential for deterioration secondary to development of ipsilateral
hindfoot arthritis. Cuester et al provided long term followup (mean
22 yrs) of patients who underwent ankle fusion and noted that
majority of patients had substantial and accelerated arthritic
changes in the ipsilateral foot but not the knee.

93-A 65-year-old man complains of ankle pain refractory to


bracing, physical therapy and NSAIDS. A radiograph is
provided in Figure A. He is treated with ankle arthroplasty but
continues to have pain and limited ambulation 10 months
following surgery. Physical exam is notable for well healed
incisions and no instability with anterior drawer and inversion
testing. A current radiograph is provided in Figure B. Which
of the following is the most likely cause of the continued
pain?

FIGURES: A B
Dr.Wael Abboud (Foot and Ankle OB)
Dr.Wael Abboud (Foot and Ankle OB)

1. Improper surgical procedure


2. Component loosening due to polyethylene wear
3. Lateral ligament failure
4. Syndesmotic nonunion
5. It is normal to have continued pain at 10 months
following this surgery

PREFERRED RESPONSE ▼ 4
CORRECT

Figure A demonstrates an osteoarthritic ankle. Figure B


demonstrates a total ankle arthroplasty with nonunion of the
syndesmotic fixation above the components. Arthroplasty and
arthrodesis are both options for surgical treatment of osteoarthritis.
Contraindications to arthroplasty include heavy laborers, Charcot
joint, neurologic muscle imbalance, severe malalignment, and/or
uncorrectable ankle instability. This patient is struggling with pain
due to nonunion of the syndesmotic fixation. Illustration A
demonstrates a well-healed syndesmotic fixation above a total
ankle replacement for comparison. Syndesmotic nonunion
increases the risk of tibial component loosening. Pyevich et al
reviewed mid-term (2-12 years) follow-up of 86 total ankle
arthroplasties. Five ankles (6 percent) had been revised for
component loosening. Delayed union and nonunion of the
syndesmosis was associated with revision. In a review article,
Alvine emphasizes proper patient selection and careful
postoperative rehabilitation to achieve long-term success with
ankle arthroplasty.
Dr.Wael Abboud (Foot and Ankle OB)

94-All of the following have been recognized as risk factors


for nonunion or delayed union following subtalar arthrodesis
EXCEPT:

QID: 643

1. smoking
2. failure of previous subtalar arthrodesis
3. more than 2 millimeters of avascular bone at arthrodesis
site
4. prior ipsilateral tibiotalar arthrodesis
5. use of autograft

PREFERRED RESPONSE ▼ 5
CORRECT
Dr.Wael Abboud (Foot and Ankle OB)

Easley et al reported on a series of isolated subtalar arthrodeses in


adults. Their results suggest that the outcome following isolated
subtalar arthrodesis is not as favorable as has been reported in
previous studies. The rate of union was significantly diminished by
smoking, the presence of more than two millimeters of avascular
bone at the arthrodesis site, and the failure of a previous subtalar
arthrodesis (p < 0.05 for all). Other factors that probably affect the
union rate include the use of structural allograft and performance
of the arthrodesis adjacent to the site of a previous ankle
arthrodesis.

95-Figure A is a dissection of the medial aspect of the left


ankle and foot. Which of the following nerves indicated in
Figure A is most commonly implicated in nerve entrapments
in the running athlete?

FIGURES: A
Dr.Wael Abboud (Foot and Ankle OB)

1. A
2. B
3. C
4. D
5. E

PREFERRED RESPONSE ▼ 1
CORRECT

Point A demonstrates the first branch of the lateral plantar nerve


(Baxter's nerve). Baxter's nerve innervates the abductor digiti
quinti, quadratus plantae, and flexor digitorum brevis muscles. It
traverses just superior to the insertion of the plantar fascia on the
medial calcaneal tuberosity. Compression of this nerve causes
maximal pain over the plantar medial aspect of the foot, and can
be difficult to distinguish from plantar fasciitis. Baxter's nerve
compression is a common pattern of entrapment found in the
running athlete. Illustration A depicts the sensory distribution of the
tibial nerve. Illustration B shows the dissection with the other
responses labeled; Point B are the medial calcaneal nerve
branches, Point C is the tibial nerve, Point D is the lateral plantar
nerve and Point E is the medial plantar nerve.

The review article by Neufeld and Cerrato discusses plantar


fasciitis and its differential diagnoses including Baxter's nerve
compression, calcaneal stress fracture, lumbar radiculopathy, and
plantar fibromatosis.
Dr.Wael Abboud (Foot and Ankle OB)
Dr.Wael Abboud (Foot and Ankle OB)

96-A 40-year-old runner complains of heel pain for 4 months.


He reports tenderness over the abductor hallucis origin with a
positive Tinel's sign radiating to the lateral foot. The pain
worsens with prolonged activity. What is the most likely
diagnosis?

QID: 406

1. plantar fasciitis
2. heel fat pad fat atrophy
3. compression of the first branch of the lateral plantar
nerve (Baxter's nerve)
4. achilles tendinitis
5. tarsal tunnel syndrome
Dr.Wael Abboud (Foot and Ankle OB)

PREFERRED RESPONSE ▼ 3
CORRECT

Patients with distal entrapment of the lateral plantar nerve or its


branches usually present with chronic heel pain that has been
present for 9-12 months. Many of their symptoms are similar to
plantar fasciitis, especially the location of their pain and their
startup pain. In addition to the mechanical symptoms of plantar
fasciitis, they present with neuritic pain that is unrelated to weight
bearing or loading of the foot. Baxter and Thigpen described a
biomechanical basis for the entrapment of the first branch of the
lateral plantar nerve in the athlete. They proposed that entrapment
results from the stretching and tethering of the plantar nerves,
which are encased in the abductor hallucis deep fascial leashes,
and from the hypertrophy of the small foot muscles, as well as
from the increased forces in the hindfoot in the running athlete that
create additional microtrauma to the runner's medial heel
structures. They also noted that most of their patients with sports-
related injuries had a normally arched or cavus-type foot.

97-Which nerve innervates the abductor digiti quinti and can


be compressed as it travels under the fascia of the abductor
hallucis muscle leading to symptoms of plantar heel pain?

QID: 811

1. medial plantar
2. medial calcaneal
3. sural
4. superficial peroneal
5. first branch of the lateral plantar nerve

PREFERRED RESPONSE ▼ 5
CORRECT

Most of the plantar aspect of the foot is derives its sensory


innervation from the plantar nerves, which are the terminal sensory
branches of the tibial nerve. The lateral plantar nerve innervates
the abductor digiti quinti and travels across the heel, just anterior
Dr.Wael Abboud (Foot and Ankle OB)

to the medial tuberosity of the calcaneus. The first branch of the


lateral plantar nerve passes between the deep taut fascia of the
abductor hallucis muscle and the medial caudal margin of the
medial head of the quadratus plantae muscle. The superfical
branch of the peroneal nerve supplies the dorsum of the foot
except the first web space (deep branch). The sural nerve supplies
that lateral aspect of the foot and not the plantar surface. Baxter et
al showed good to excellent results in 89% of patients where they
decompressed the first branch of the lateral plantar nerve for
medial heel pain.

98-A 55-year-old female with longstanding type I diabetes


presents for evaluation of her left foot, which is shown in
Figure A. On exam, the soft tissue infection extends to the
metatarsals plantarly, and there is a palpable posterior tibial
artery pulse. She is otherwise medically stable, and without
sepsis. Which of the following treatment options will most
likely result in definitive management of her forefoot
gangrene and allow the highest level of functional activity
after surgery?

FIGURES: A
Dr.Wael Abboud (Foot and Ankle OB)

1. Transmetatarsal amputation
2. Below the knee amputation
3. Syme amputation
4. Above the knee amputation
5. Extensive soft-tissue debridement, local wound care,
and antibiotic therapy

PREFERRED RESPONSE ▼ 3
CORRECT

Isolated forefoot gangrene in the presence of a palpable posterior


tibial artery pulse can be definitively managed with a Syme
amputation, which leads to a relatively high functional status in
these patients. A Syme amputation includes ankle disarticulation,
removal of malleoli, and anchoring heel pad to the weight bearing
surface. A viable heel pad is critical for surgical success of a Syme
Dr.Wael Abboud (Foot and Ankle OB)

amputation, and it receives its blood supply from branches of the


posterior tibial artery. An example of this post-operatively is shown
in Illustration A.

Francis et al reviewed the charts of 26 dysvascular patients with


forefoot necrosis who underwent Syme amputation. They found
that 85% of the patients with a palpable posterior tibial pulse had a
successful amputation in contrast to one out of four who did not
have a palpable pulse before surgery. They concluded that the
single most important feature for success with Syme amputations
is to limit the operation to those patients with a palpable posterior
tibial pulse before surgery.

Laughlin et al reviewed the surgical results and functional outcome


of 52 patients treated with Syme amputations for forefoot
gangrene. They found that 90% of the patients who had a posterior
tibial artery with either a triphasic waveform or a normal pulse
achieved a healed wound suitable for prosthetic wear after
undergoing a Syme amputation. This is compared to only a 57%
success rate in patients with compromised posterior tibial arterial
flow.

Incorrect Answers:
1-A transmetatarsal amputation may be used initially to clear an
infection before completing a more proximal ampuation. However,
this would not be appropriate as definitive management due to its
proximity to the infected and necrotic tissue distally.
2,4-An above or below knee amputation in the presence of a
palpable posterior tibial artery would not be appropriate as
significantly better functional results result from a more distal Syme
amputation.
5-Soft tissue debridement, local wound care, and antibiotic therapy
would not definitively treat forefoot gangrene.
Dr.Wael Abboud (Foot and Ankle OB)

99-A 44-year-old male with long standing insulin dependent


diabetes complains of a non-healing plantar foot ulcer. The
ulcer is shown in Figure A. The second metatarsal head can
be probed at the base of the wound, and he lacks plantar
sensation. Laboratory work-up for infection is negative.
Which of the following is the best initial treatment?

FIGURES: A
Dr.Wael Abboud (Foot and Ankle OB)

1. Ray resection and primary wound closure


2. Oral antibiotics and local wound care
3. Local wound care and non-weight bearing in a
removable boot
4. Surgical debridement, dressing changes, and antibiotics
5. MRI of the foot to evaluate for underlying osteomyelitis

PREFERRED RESPONSE ▼ 4
CORRECT

The ability to probe bone at the base of the ulcer is indicative of


underlying osteomyelitis and this should be initially treated with
surgical debridement, IV antibiotics and local wound care. Ray
resection or partial foot amputation could be considered in this
case, however this is usually reserved for patients who have failed
local treatment or are systemically ill from their ulcer. Oral
antibiotics or boot application are not an aggressive enough
treatment option in this clinical scenario. Underlying osteomyelitis
should be assumed to be present in this case; therefore, an MRI is
Dr.Wael Abboud (Foot and Ankle OB)

not useful in guiding treatment at this stage. Pinzur et al provide an


overview of diabetic foot care and address physical examination,
patient education, and basic treatment guidelines. Grayson et al
evaluated 76 foot ulcers and found that palpating bone on probing
the pedal ulcer had a sensitivity of 66% for osteomyelitis, a
specificity of 85%, and should be should be included in the initial
assessment of all diabetic patients with infected pedal ulcers.

100-A 65-year-old diabetic male presents with the foot ulcer


shown in Figure A. There is no exposed bone, and no signs of
infection. Pulses are palpable. What additional information
should be obtained next to help guide this patient's
treatment?

FIGURES: A

1. MRI scan with contrast


2. Ankle-brachial index
Dr.Wael Abboud (Foot and Ankle OB)

3. Results of Silverskiold test


4. Transcutaneous oxygen measurements of the toes
5. Hemoglobin A1C level

PREFERRED RESPONSE ▼ 3
CORRECT

Forefoot ulcers are exacerbated by a fixed plantarflexion


contracture secondary to either a tight Achilles or gastrocnemius
tendon. The Silfverskiold test differentiates isolated contractures of
the gastrocnemius from the gastrocsoleus complex. The forefoot is
inverted and the hind foot is positioned in subtalar neutral to lock
the transverse tarsal joints. The knee is first flexed with ankle
dorsiflexion and then compared to passive motion with the knee
extended. Illustration A shows a positive test with equinus
contracture In the presence of palpable pulses and a plantarflexion
contracture.

Wagner grade 1 and 2 ulcers (abscence of osteomyelitis) should


be treated with total contact casting AND gastrocnemius recession
when indicated to decrease the risk of ulcer recurrence. An MRI
scan with contrast would be helpful if there was concern for
infection. Ankle-brachial index and transcutaneous oxygen
measurements should be performed in the absence of palpable
pulses. HgbA1C levels are useful in guiding the chronic
management of diabetes and should be optimized. However, it is
less useful in the acute management of a plantar ulcer.

Lin et al looked at 93 neuropathic diabetes mellitus patients with


foot ulcers who underwent a total contact cast protocol. Fifteen of
the patients showed delayed ulcer healing and all were noted to
have an ankle equinus deformity and limited joint motion. This
group was treated with percutaneous tendo-Achilles lengthening,
and all but one ulcer went on to heal.

Mueller et al randomized 64 subjects into two treatment groups,


immobilization in a total-contact cast alone or combined with
percutaneous Achilles tendon lengthening. All ulcers healed in the
Achilles tendon lengthening group, and the risk for ulcer
recurrence was 75% less at seven months and 52% less at two
years than the risk of recurrence in the total-contact cast alone
group.
Dr.Wael Abboud (Foot and Ankle OB)

101-A 44-year-old man with diabetes mellitus has a non-


healing Wagner grade 1 ulcer shown in Figure A for the past 8
months. Conservative management with total contact casting
has not resolved the ulcer. Physical examination reveals loss
of protective sensation by Semmes-Weinstein testing, no
signs of infection, positive Silfverskiold test indicating
gastrocnemius contracture, and palpable pedal pulses. What
is the next most appropriate step in management?

FIGURES: A
Dr.Wael Abboud (Foot and Ankle OB)

1. Integra artificial dermis placement followed by split


thickness skin grafting
2. Continued total contact casting
3. Split thickness skin grafting to ulcer
4. Strayer procedure (gastrocnemius lengthening)
5. Weil metatarsal decompression osteotomy

PREFERRED RESPONSE ▼ 4
CORRECT

Diabetic forefoot ulcers can be refractory to conservative


management due to a fixed plantarflexion contracture and can be
corrected with a Strayer procedure.

The Silverskiold test differentiates isolated contractures of the


gastrocnemius from the gastrocsoleus complex. The forefoot is
inverted and the hind foot is positioned in subtalar neutral to lock
the transverse tarsal joints. The knee is first flexed with ankle
dorsiflexion and then compared to passive motion with the knee
extended. Isolated gastrocnemius contracture is present if
dorsiflexion is increased during knee flexion compared to knee
extension and indicates that an isolated gastrocnemius fascia
lengthening (Strayer procedure) is sufficient. If there is an equinus
contracture that does not improve with knee flexion then the entire
gastrocsoleus complex is contracted and an achilles tendon
lengthening (Hoke procedure) is required and not an isolated
gastrocnemius facia lengthening (Strayer procedure).

Mueller et al randomized 64 subjects into two treatment groups,


immobilization in a total-contact cast alone or combined with
percutaneous Achilles tendon lengthening. All ulcers healed in the
Achilles tendon lengthening group, and the risk for ulcer
recurrence was 52% less at 2 years compared to treatment with
total-contact casting alone.

Lin et al evaluated 93 neuropathic diabetic patients with foot ulcers


who underwent conservative management. Fifteen of the patients
showed delayed ulcer healing and all were noted to have an ankle
plantarflexion contracture. This group was treated with
percutaneous tendo-Achilles lengthening, and 14 went on to heal
the ulcer.
Dr.Wael Abboud (Foot and Ankle OB)

102-A 65-year-old patient with poorly controlled diabetes


develops 2nd metatarsal head osteomyelitis deep to a
superficial ulcer. Which nonoperative treatment modality
would have the highest chance of success?

QID: 2917

1. IV Vancomycin
2. Oral Levofloxacin
3. IV antibiotics based on ulcer swab culture sensitivity
4. IV antibiotics based on percutaneous bone biopsy
culture sensitivity
5. Elevation and non-weight bearing

PREFERRED RESPONSE ▼ 4
CORRECT

Intravenous antibiotics tailored to bone biopsy culture sensitivities


have the best chance of successful treatment of foot osteomyelitis
in diabetics. A multi-center retrospective review by Senneville et al.
demonstrated only (64%) remission of foot osteomyelitis in
diabetics at one year. Antibiotic tailored by bone biopsy culture
sensitivities was the only factor that significantly affected remission
rates of osteomyelitis.

103-A 57-year-old man taking metformin for diabetes and


gabapentin for peripheral neuropathy has a superficial plantar
midfoot ulcer with a clean, noninfected appearance. Total
contact casting is implemented for mechanical relief. Which
of the following radiographs most likely corresponds to the
clinical situation described.

FIGURES: A B C D E
Dr.Wael Abboud (Foot and Ankle OB)
Dr.Wael Abboud (Foot and Ankle OB)
Dr.Wael Abboud (Foot and Ankle OB)
Dr.Wael Abboud (Foot and Ankle OB)
Dr.Wael Abboud (Foot and Ankle OB)

1. Figure A
2. Figure B
3. Figure C
4. Figure D
5. Figure E

PREFERRED RESPONSE ▼ 3
CORRECT

This diabetic patient with a plantar midfoot ulcer most likely has
Charcot arthropathy of the foot. This is shown radiographically in
Figure C as evident by the midfoot destruction and joint
subluxation. Charcot arthropathy occures in 7.5% of neuropathic
diabetics. Wukich et al described an 83% rate of ulcer healing with
total contact casting in neuropathic ulcers and noted a 17% rate of
complications (most being due to skin irritation from the cast).
Figure A shows a homolateral Lisfranc injury and Figure B shows a
hallux valgus deformity. Figure D shows a radiograph of a cavus
foot often associated with Charcot Marie Tooth disease. Figure E
shows a radiograph of an acquired flatfoot deformity with midfoot
subluxation but there is absent fragmentation, osteopenia, or bony
destruction indicating Charcot arthropathy of the foot.

104-You are caring for a 72-year-old male with diabetes and


peripheral neuropathy with a non-healing forefoot ulcer
(Figure A). To reduce the plantar pressure on his forefoot,
which of the following shoe modifications would you
suggest?

FIGURES: A
Dr.Wael Abboud (Foot and Ankle OB)

1. Polyethylene foam insole


2. Open toe sandles
3. Rocker sole shoes
4. Custom indepth shoes
5. Hard postoperative shoe

PREFERRED RESPONSE ▼ 3
CORRECT

The rocker sole best reduces forefoot plantar pressure. Brown et


al. examined the effect of different rocker soles on plantar pressure
in 40 patients without foot deformities by measuring the plantar
pressures over a 400 step course. All 3 rocker designs showed a
significant reduction in peak pressure and the pressure time
integral. Janisse and Janisse review the various shoe
modifications in the non-operative treatment of foot and ankle
pathology, and review the treatment options regarding rocker sole
shoes, and other shoe modifications.
Dr.Wael Abboud (Foot and Ankle OB)

105-A 45-year-old diabetic male has a Wagner type 3 heel


ulcer shown in Figure A that measures 4x2cm and is
recalcitrant to debridements and total contact casting for 4
months. The patient has palpable pulses, active drainage at
the ulcer, and does not have protective sensation with a 5.07
Semmes-Weinstein filament. Radiograph and MRI (sagittal
and axial) images are shown in Figures B-D respectively. In
addition to bone culture biopsy, debridement and antibiotic
therapy, what surgical intervention is most appropriate?

FIGURES: A B C D
Dr.Wael Abboud (Foot and Ankle OB)
Dr.Wael Abboud (Foot and Ankle OB)
Dr.Wael Abboud (Foot and Ankle OB)

1. Ankle disarticulation
2. Soft tissue fasciocutaneous flap coverage
3. Partial calcanectomy
4. Below knee amputation
5. Soft tissue free flap coverage

PREFERRED RESPONSE ▼ 3
CORRECT

This patient has failed conservative management and has


evidence of osteomyelitis on MRI and bone scan. Intravenous
antibiotics tailored to bone culture biopsy sensitivities have the
best chance of successful treatment of foot osteomyelitis in
diabetics. A partial calcanectomy would be the next most
appropriate step in management if debridement was not sufficient.
Partial or complete calcanectomy are preferred over higher level
amputations such as Syme ankle disarticulations and transtibial
Dr.Wael Abboud (Foot and Ankle OB)

amputations to preserve limb length and decrease morbidity. Soft


tissue coverage can be appropriate for ulcers in the absence of
infection and higher level amputations would be more appropriate
if there was evidence of a dysvascular limb or inability to heal the
ulcer once the tissues are free of infection.

The article by Smith et al reviewed 12 patients with large heel


ulcers and calcaneal osteomyelitis and found the wound healed
after partial calcanectomy in ten of the twelve patients. Illustration
A depicts a proposed line of resection for a partial calcanectomy
as described by Smith et al. They note that the achilles must he
released and allowed to retract proximally since the posterior
process of the calcaneus is removed, with the cut started one
centimeter posterior to the edge of the subtalar and
calcaneocuboid joint. A custom molded orthosis is needed
postoperatively to help cushion the heel during ambulation. A full
listing of the Wagner classification is shown in Illustration B.
Dr.Wael Abboud (Foot and Ankle OB)
Dr.Wael Abboud (Foot and Ankle OB)

106-All of the following are predictive of poor healing of


diabetic foot ulcers EXCEPT:

QID: 786

1. transcutaneous oxygen pressure < 20 mmHg


2. systolic blood pressure > 140 mmHg
3. ankle-brachial index < 0.45
4. albumin < 3.5 g/dL
5. total lympocyte count < 1,500/mm3

PREFERRED RESPONSE ▼ 2
CORRECT

In evaluation of a non-healing diabetic foot ulcer the patient's


capacity for healing can be assessed with several methods.
Vascular evaluation begins with noninvasive testing that includes a
ankle-brachial index(ABI) and transcutaneous oxygen pressure.
An ABI of < 0.45 or transcutaneous oxygen pressure of < 20
mmHg are negative predictors of healing. Laboratory studies help
assess immunity and overall nutrition. An albumin of < 3.5 g/dL or
a total lymphocyte count of < 1,500/mm3 are negative predictors of
healing. Systolic blood pressure has not been shown to be
predictive.

107-Which of the following patients with type 2 diabetes


mellitus is most likely to develop a foot ulcer?

QID: 165

1. 54-year-old female unable to feel the presence of a 5.07


Semmes-Weinstein monofilament on the plantar aspect of the
foot
2. 63-year-old male with transcutaneous oxygen pressures
(TcpO2) of 30 mm Hg
3. 51-year-old male with ratio of ankle to brachial pressures
Dr.Wael Abboud (Foot and Ankle OB)

of < 0.6
4. 71-year-old male with serum albumin of 3.1 g/dL
5. 60-year-old with autonomic dysfunction leads to drying
of skin due to lack of normal glandular function

PREFERRED RESPONSE ▼ 1
CORRECT

The primary risk factor for the development of a diabetic foot ulcer
is loss of protective sensation and this is commonly tested with a
5.07 Semmes-Weinstein monofilament. Once an ulcer is present,
non-invasive vascular evaluation is performed to determine ulcer
healing potential via ankle-brachial index(ABI) or transcutaneous
oxygen pressure (TcpO2). An ABI of < 0.45 or transcutaneous
oxygen pressure of < 20 mmHg are negative predictors of healing.
Laboratory studies help assess immunity and overall nutrition. An
albumin of < 3.5 g/dL or a total lymphocyte count of < 1,500/mm3
are negative predictors of diabetic ulcer healing. Foot ulcers are
considered the most likely predictor of eventual lower extremity
amputation in patients with diabetes mellitus.

108-The United States Therapeutic Shoe Bill ensures that


Medicare will cover all of the following EXCEPT:

QID: 303

1. Custom molded shoes


2. Shoe wedges
3. Inserts for missing toes
4. Charcot restraint orthotic walker boot
5. Velcro closure shoe modification

PREFERRED RESPONSE ▼ 4
CORRECT

Diabetes results in a 15 to 40 fold increased risk of amputation. An


estimated 67,000 diabetes related amputations occur in the U.S.
each year. The Therapeutic Shoe Bill (TSB) covers 4 things:
Dr.Wael Abboud (Foot and Ankle OB)

Custom-molded shoes, Depth shoes (3/16 inch extra depth


minimum), inserts, & shoe modifications (ie: rocker soles,
metatarsal bars, wedges, flared heels, Velcro closures, and inserts
for missing toes). The TSB provides coverage for one of the
following within one calendar year: One pair of custom-molded
shoes (including one pair of inserts provided with such shoes) and
two additional pairs of inserts; or one pair of depth shoes and three
pairs of inserts. The Charcot Restraint orthotic walker boot is not
covered under this particular bill, but may be covered under
Medicare (and other plans) if deemed medically necessary.

109-A 62-year-old diabetic female presents with a Wagner


grade 1 foot ulcer. Upon examination of the foot, no dorsalis
pedis pulse is palpable. Each of the following noninvasive
vascular tests indicate a good prognosis for ulcer healing
EXCEPT:

QID: 344

1. Triphasic waveforms
2. Ankle-brachial indices (ABI) of 0.72
3. Absolute toe pressure of 45 mm Hg
4. Transcutaneous oxygen measurements (pO2) of 25mm
Hg
5. Presence of hair on the toes

PREFERRED RESPONSE ▼ 4
CORRECT

Approximately 60-70% of patients who have had diabetes for over


10 years have evidence of peripheral vascular disease. Vascular
examination of the diabetic foot includes evaluating for the
presence of dorsalis pedis and posterior tibial artery pulses. The
absence of hair on the foot is an additional indicator of
compromised blood flow. Transcutaneous oxygen measurement
>30 mm Hg indicates that blood flow is adequate for healing. An
ABI =0.45 is needed to heal an ulcer in the diabetic foot. It is
important to note that peripheral vascular calcifications can falsely
elevate ABI results, thus reducing the sensitivity of ABI's as a
Dr.Wael Abboud (Foot and Ankle OB)

screening test. Toe pressures >40 mm Hg have been associated


with ulcer healing. The article by Wagner is a review article
discussing the evaluation of foot vascularity and establishing the
Wagner classification of foot ulcers. The ulcers are categorized
into six grades ranging from grade zero to grade five as seen in
Illustration A. The grade is determined based on depth of the skin
lesion and the presence or absence of infection and gangrene. A
Wagner 1 ulcer as found in this case is a superficial or full
thickness ulcer that does not expose ligaments, deep fascia, or
bone.

110-All of the following are predictors of future amputation in


diabetic patients EXCEPT?

QID: 235
Dr.Wael Abboud (Foot and Ankle OB)

1. Diabetic foot ulceration


2. Neuropathy
3. Infection
4. Gangrene
5. Increased ankle-brachial index

PREFERRED RESPONSE ▼ 5
CORRECT

The references state that diabetic patients who underwent


amputations also had concurrent: Ulceration 84%, Neuropathy
64%, Infection 59%, Gangrene 55%, and Ischemia 46%. Other risk
factors factors included elevated hemoglobin A1c level, decreased
oxygen tension levels, and a decreased ankle-brachial index.

111-A 34-year-old patient is noted to have a lack of ankle


dorsiflexion by 5 degrees with knee extension as seen in
Figure A. However, the ankle dorsiflexion improves to 20
degrees with knee flexion as seen in Figure B. Which of the
following diagnoses would benefit MOST from a
gastrocnemius recession (Strayer procedure)?

FIGURES: A B
Dr.Wael Abboud (Foot and Ankle OB)

1. Chronic insertional achilles tendonitis


2. Freiberg's infraction
Dr.Wael Abboud (Foot and Ankle OB)

3. Chronic retrocalcaneal bursitis


4. Chronic calcaneal osteomyelitis
5. Chronic plantar forefoot ulcer

PREFERRED RESPONSE ▼ 5
CORRECT

This patient has a positive Silverskiold test. The Silfverskiold test


differentiates isolated contractures of the gastrocnemius from the
gastrocsoleus complex. The forefoot is inverted and the hind foot
is positioned in subtalar neutral to lock the transverse tarsal joints.
The knee is first flexed with ankle dorsiflexion and then compared
to passive ankle dorsiflexion with the knee extended. Isolated
gastrocnemius contracture is present if dorsiflexion is increased
during knee flexion compared to knee extension and indicates that
an isolated gastrocnemius fascia lengthening (Strayer procedure)
is sufficient. If there is an equinus contracture that does not
improve with knee flexion then the entire gastrocsoleus complex is
contracted and an achilles tendon lengthening (Hoke procedure) is
required.

The study by Armstrong et al found that a percutaneous achilles


tendon lengthening led to a 25% decrease in contact peak
pressure over the metatarsal heads in 10 diabetic patients with
prior ulceration and high risk for reulceration.

AAOS Comprehensive Orthopaedic Review additionally states that


"lengthening can be helpful in reducing plantar forefoot pressure"
and ostectomy or realignment arthrodeses can also be helpful in
managing deformities associated with diabetic foot ulcers.

112-A 37-year-old man with type-1 diabetes mellitus reports a


3-month history of a plantar foot ulcer shown in Figure A. His
pulses are palpable and sensation to a 5.07 Semmes-
Weinstein monofilament is absent on the entire plantar
surface of the foot. There is no erythema or drainage and
there is no bone encountered during probing of the ulcer.
There is no fever and the white blood cell count is normal.
The C-reactive protein and erythrocyte sedimentation rate are
normal. What is the most appropriate next step in treatment?
Dr.Wael Abboud (Foot and Ankle OB)

FIGURES: A

1. Total contact casting and empiric IV antibiotics


2. Surgical debridement, dressing changes, and empiric IV
antibiotics
3. Charcot restraint orthotic walker (CROW)
4. Surgical debridement, dressing changes, biopsy, and
culture specific IV antibiotics
5. Total contact casting

PREFERRED RESPONSE ▼ 5
CORRECT

Total contact casting is an effective therapy for healing chronic


neuropathic plantar ulcers in individuals with diabetes mellitus and
other chronic sensory neuropathies. There is no sign of clinical
infection present so surgical debridement and antibiotics are not
indicated. Total contact casts are snug-fitting, below-knee casts
that protect insensate limbs from repetitive trauma, promote ulcer
healing, and allow the patient to remain ambulatory status. Most
ulcers recur within the first 6 months (typically within the 1st
Dr.Wael Abboud (Foot and Ankle OB)

month) after initial healing. When the active disease phase is


complete, the patient can be fitted with a Charcot restraint orthotic
walker (CROW), later followed by a custom shoe with orthoses.

113-A 50-year-old male with long-standing type 1 diabetes


presents with redness, swelling and crepitus in his foot two
weeks after a twisting injury. He denies significant pain.
Elevation of the extremity reduces the hyperemia and CBC,
ESR, and C-reactive protein levels are normal. A radiograph is
shown in Figure A. What is the most likely diagnosis?

FIGURES: A

1. osteomyelitis
2. cellulitis
3. fracture
4. charcot neuropathy
5. diabetic foot ulcer
Dr.Wael Abboud (Foot and Ankle OB)

PREFERRED RESPONSE ▼ 4
CORRECT

This is a classic presentation of Charcot arthropathy in a diabetic


patient after sustaining a relatively minor trauma. Infection is a
possibility but unlikely given the normal laboratory values. In
addition, hyperemia that regresses with elevation is classic for
Charcot and not consistent with infection. Neuropathy has the
greatest affect on diabetic foot pathology and the most sensitive
test is the Semme's Weinstein monafilament testing. Guyton et al's
ICL on the diabetic foot presents a comprehensive review of this
topic.

114-You are seeing a 62-year-old male for ankle and foot


swelling (Figures A-C). There is no history of trauma and he
has never seen a physician before. In addition to his lower
extremity care, what other medical condition should he be
evaluated for?

FIGURES: A B C
Dr.Wael Abboud (Foot and Ankle OB)
Dr.Wael Abboud (Foot and Ankle OB)

1. Cardiovascular disease
2. Hypertension
3. Diabetes mellitus
4. Spinal stenosis
5. Rheumatoid arthritis

PREFERRED RESPONSE ▼ 3
CORRECT

This patient is presenting with charcot arthropathy, a known


complication of diabetes mellitus. Clinical photograph and
radiographs show the characteristic appearance of the charcot foot
with complete arch collapse and multi-joint end stage degenerative
joint disease through the entire hind, mid, and fore-foot. As
described by Guyton and Saltzman, there is a complex
pathophysiology by which diabetes contributes to the foot
deformities seen in charcot arthropathy. This pathophysiology
Dr.Wael Abboud (Foot and Ankle OB)

includes alterations in peripheral nerves, bones/soft tissue, gait


kinematics, microscopic/macroscopic vascularity, immune system,
and mechanisms of wound healing. To treat, one must address the
mechanical and biologic aspects of the disease.

115-A 62-year-old gentleman with a 10-year history of Type II


diabetes complains of warmth, swelling, and pain in his right
foot that has progressively worsened over the past 6 weeks.
He denies fevers or chills, and states that the swelling and
warmth dissipates each night after he sleeps with his foot
elevated on pillows. A clinical photograph of the foot is
provided in Figure A. The midfoot is hot to touch and mildly
tender with palpation. A radiograph is provided in Figure B.
Which of the following is the most appropriate management?

FIGURES: A B
Dr.Wael Abboud (Foot and Ankle OB)

1. Custom orthotics with first ray recession and lateral heel


posting
2. Total contact cast and non-weight bearing
3. Intravenous antibiotics
Dr.Wael Abboud (Foot and Ankle OB)

4. Talonavicular and tarsometarsal arthrodeses


5. Transtibial amputation

PREFERRED RESPONSE ▼ 2
CORRECT

The clinical presentation, photograph, and radiograph are


consistent with diabetic charcot neuropathy of the midfoot. The
lack of systemic symptoms and resolution of erythema with foot
elevation rule against the presence of infection. This patient
appears to be in the fragmentation phase of the pathologic process
given the osteopenia, fracture, and collapse of the midfoot. This is
followed by the coalescence and reconstitution phases. The goal
of treatment in the first phase is to prevent further collapse and
deformity. Given there is no ulceration, and/or deep infection
present, this is best accomplished through protected weight
bearing and total contact casting. A CROW walker would could
also be considered. Examples of total contact casting and a
CROW walker are demonstrated in Illustrations A and B. The
objective is to prevent high contact stress points and subsequent
ulceration in the insensate foot. Surgical intervention should be
reserved for debridement of a deep infection from open wounds or
arthrodesis of arthritic joints following the consolidation that occurs
during the reconstitution phase.
Dr.Wael Abboud (Foot and Ankle OB)
Dr.Wael Abboud (Foot and Ankle OB)

116-A 65-year-old diabetic female presents with a nine-month


history of mild ankle pain and instability. She denies any
specific injury and she does not have any foot ulcerations or
wounds. Her ESR, CRP, and WBC levels are within normal
limits. Her radiographs are shown in Figures A and B. What is
the most appropriate initial treatment?

FIGURES: A B
Dr.Wael Abboud (Foot and Ankle OB)
Dr.Wael Abboud (Foot and Ankle OB)

1. Modification of shoe wear


2. Use of a CROW walker boot
3. Ankle arthrodesis
4. Spanning external fixation of the ankle and hindfoot
5. Below-knee amputation

PREFERRED RESPONSE ▼ 2
CORRECT

Figures A and B show severe Charcot changes to the hindfoot,


leading to the patient's reports of pain and instability. Initial
treatment should include bracing and frequent skin checks to
monitor for development of ulcerations or other skin lesions.
Neuropathic osteoarthropathy, otherwise known as Charcot
neuroarthropathy, is a chronic, degenerative arthropathy and is
associated with decreased sensory innervation. Typical findings
include joint destruction, disorganization, and effusion with
Dr.Wael Abboud (Foot and Ankle OB)

osseous debris. Progression of Charcot neuroarthropathy often


follows a predictable clinical and radiographic pattern and is
described by the Eichenholtz classification found in Illustration A.

Hatzis et al reviewed a case series of neuroarthropathy of the


shoulder (radiograph shown in Illustration B), and found that
syringomyelia is the most common etiology of this disorder in the
shoulder.

The referenced article by van der Ven is a review of the etiologies,


pathogenesis, treatments, and outcomes of Charcot
neuroarthropathy of the foot and ankle.
Dr.Wael Abboud (Foot and Ankle OB)

117-A 57-year-old woman with type 2 diabetes has had the


plantar foot ulcer shown in Figure A for 2 months. Medical
comorbidities include renal insufficiency and hypertension. A
Dr.Wael Abboud (Foot and Ankle OB)

radiograph is provided in Figure B. Following ulcer


debridement, what initial management will provide the best
long term function?

FIGURES: A B
Dr.Wael Abboud (Foot and Ankle OB)
Dr.Wael Abboud (Foot and Ankle OB)

1. Carbon fiber shank insole


2. Charcot restraint orthotic walker (CROW)
3. AFO (ankle foot orthosis) with posterior leaf spring
4. Total contact casting
5. Accomodative plastizote insole with depression cut into
the midfoot and extra-depth shoes

PREFERRED RESPONSE ▼ 4
CORRECT

Initial treatment of Charcot arthropathy includes a total contact cast


(TCC) continued for up to 4 months. When the active disease
phase has ended, the patient can be fitted with a CROW (charcot
restraint orthotic walker) and, later with a custom shoe with
orthoses. Charcot arthropathy is a destructive process, most
commonly affecting joints of the foot and ankle in diabetics with
peripheral neuropathy. Affected individuals present with swelling,
warmth, and erythema, often without history of trauma. Bony
fragmentation, fracture, and dislocation progress to foot deformity,
bony prominence, and instability. Treatment is focused on
providing a stable and plantigrade foot for functional ambulation
with accommodative footwear and orthoses. The Charcot Restraint
Orthotic Walker (CROW) seen in Illustration A is a custom,
bivalved, total contact AFO with full foot enclosure, rigid rocker
sole, and custom insole. The CROW usually is used after an initial
period in a TCC to enable reduction of edema before fabrication of
the CROW. The total contact design simulates a TCC, but the
CROW allows better hygiene and comfort because it is removable.
The CROW controls edema, enables ambulation, and prevents
deformity as coalescence proceeds. Disadvantages of the CROW
include the high costs of fabrication and maintenance.
Dr.Wael Abboud (Foot and Ankle OB)

118-A 29-year-old male presents with left knee instability and


progressive gait disturbance. He is only able to ambulate with
the assistance of crutches or a walker. He has no pain with
ambulation and has decreased vibratory sensation in the
bilateral lower extremities. Radiographs are shown in Figures
A-B. All of the following are likely etiologies for his
presentation EXCEPT:

FIGURES: A B
Dr.Wael Abboud (Foot and Ankle OB)
Dr.Wael Abboud (Foot and Ankle OB)

1. Diabetes mellitus
2. Syringomyelia
3. Leprosy
4. Neurosyphilis
5. Reiter's syndrome

PREFERRED RESPONSE ▼ 5
CORRECT

Reiter's syndrome consists of a classic triad of reactive arthritis,


nongonococcal urethritis, and conjunctivitis. It is not associated
with Charcot arthropathy like the other diseases listed.
Neuropathic osteoarthropathy can be defined as bone and joint
changes that occur secondary to loss of sensation and that
accompany a variety of disorders. Charcot first described the
relationship between loss of sensation and arthropathy in 1868.
The pathophysiology of neuropathic arthropathy is debatable. The
Dr.Wael Abboud (Foot and Ankle OB)

general consensus is that the loss of proprioception and deep


sensation leads to recurrent trauma, which ultimately leads to
progressive destruction, degeneration, and disorganization of the
joint. Historically, Charcot arthropathy was considered a
contraindication to TKA and the complication rates can be
considerably higher than in non-Charcot patients.

The article by Kim et al reviewed 19 Charcot knees that underwent


TKA and found at 5 year follow-up the average HSS Knee Score
double but there was a 16% rate of loosening and 6 patients had
to undergo a fusion.

Level 4 evidency by Parvizi et al found that in 49 Charcot knees


75% required long-stem, constrained components secondary to
ligamentous instability and 75% required bone augmentation in the
form of allograft, autograft, or metal wedges. However, they found
good functional outcomes at 8 years if attention was paid to the
technical challenges found in this patient population. Illustration A
and B shows a severe case of Charcot neuroarthropathy of the
knee.
Dr.Wael Abboud (Foot and Ankle OB)
Dr.Wael Abboud (Foot and Ankle OB)

119-A 54-year-old diabetic man complains of swelling and


erythema throughout the midfoot for 2 weeks. He denies any
known trauma. The midfoot is warm, red, and swollen with no
skin disruptions on physical exam. The erythema diminishes
with elevation of the foot for 15 minutes. He has a temperature
of 100.3 degrees Fahrenheit. The patient's CRP is elevated
with a result of 6. Which of the following is the most likely
diagnosis?

QID: 1133

1. Navicular stress fracture


2. Neuropathic arthropathy
3. Osteomyelitis
Dr.Wael Abboud (Foot and Ankle OB)

4. Embolic ischemia
5. Rheumatoid arthritis

PREFERRED RESPONSE ▼ 2
CORRECT

The clinical description is most consistent with neuropathic


arthropathy (Charcot) of the midfoot. Dissipation of erythema with
elevation is the key finding differentiating neuropathic arthropathy
from infection. A foot with osteomyelitis or a deep abscess will
remain erythematous with elevation. Van der Ven et al provide a
summary of the clinical presentation, evaluation, and treatment of
Charcot arthropathy. Progression of deformity remains the
hallmark goal of treatment.

120-A newly adopted 7-year-old girl from Liberia has a history


of poliomyelitis 2 years ago. She had flaccid paralysis of her
legs that made her bedridden for 6 months. She has
recovered all strength and has no complaints at this time. The
parents would like to know if they should be concerned about
any residual effects of the infection. How should they be
counseled?

QID: 882

1. The child has no risk of further paralysis or weakness


from polio
2. The child may experience muscle weakness and fatigue
as an adult
3. If the child has no recurrent episodes of paralysis by the
time she reaches skeletal maturity, then she will never have
further weakness
4. The child will certainly have weakness and possibly
paralysis as an adult, and therefore she should abstain from
sports to prevent further muscle injury
5. The child will most likely be wheelchair bound by the
age of 25 years.
Dr.Wael Abboud (Foot and Ankle OB)

PREFERRED RESPONSE ▼ 2
CORRECT

Poliomyelitis is a viral infection that infects the central nervous


system and can cause flaccid paralysis. Post-polio syndrome may
occur in up to half of patients who have previously been afflicted
with polio. The syndrome is characterized by muscle weakness,
myalgia, and fatigue. Onset is typically 20-40 years following the
initial infection. The exact mechanism is not completely
understood. Patients exhibiting post-polio syndrome should be
counseled to get regular exercise to sub-exhaustion levels. This is
designed to keep the muscle groups toned without causing any
muscle breakdown that could potentially have deleterious long-
term effects.

 GOOD LUCK
 Best Regards

You might also like